Ch. 7, 8, 11 PATHOLOGY MIDTERM

Ace your homework & exams now with Quizwiz!

1 A 35-year-old man who received a kidney transplantation was being treated with cyclosporine, azathioprine, and high doses of corticosteroids. While on this regimen, the patient began to experience headaches and became lethargic. A clinical diagnosis of meningoencephalitis was made. He died 7 days later. Autopsy showed a gelatinous meningeal exudate, and on sectioning of the brain, multiple, small cystlike areas were seen. Microscopic examination showed areas containing rounded structures with a prominent capsule that stained brightly with mucicarmine. Which of the following tests would have been most useful for diagnosis of this condition during life? □ (A) Examination of CSF with an India ink preparation □ (B) Determination of glucose and protein content of CSF □ (C) Brain biopsy specimen stained for viral inclusions □ (D) Culture of CSF for Streptococcus pneumoniae □ (E) PCR assay to detect Epstein-Barr virus genome in lymphocytes isolated from CSF

1 (A) The patient developed cryptococcal meningoencephalitis, a complication of his immunocompromised state. Cryptococcus neoformans typically has a thick capsule, making it easily visible with the India ink preparation, a procedure that can be performed within a few minutes. The glucose and protein levels of CSF can aid in determining whether an infection is present, and what general type of organism is present, but they do not yield a specific cause. A cryptococcal antigen test also would be useful for this patient. Brain biopsies are not commonly performed, and other, less invasive methods should be pursued first. Bacterial meningitis is possible, and pneumococcus would be a common bacterial cause, but this description is consistent with cryptococcosis. PCR probes for Epstein-Barr virus are not useful in this case because acute viral meningitis usually does not cause a visible exudate, the onset of disease is more insidious, and it is not typically associated with immunocompromised states.

A 40-year-old man has a history of intravenous drug use. Physical examination shows needle tracks in his left antecubital fossa. He has mild scleral icterus. Serologic studies for HBsAg and anti-HCV are positive. He develops hepatocellular carcinoma 15 years later. Which of the following viral characteristics best explains why this patient developed hepatocellular carcinoma? □ (A) Viral integration in the vicinity of protooncogenes □ (B) Viral capture of protooncogenes from host DNA □ (C) Viral-induced injury followed by extensive hepatocyte regeneration □ (D) Viral inactivation of RB and p53 gene expression □ (E) Viral infection of inflammatory cells with host immunosuppression

1 (C) Although the hepatitis B virus (HBV) and hepatitis C virus (HCV) genomes do not encode for any transforming proteins, the regenerating hepatocytes are more likely to develop mutations, such as inactivation of p53. HBV does not have a consistent site of integration in the liver cell nuclei, and it does not contain viral oncogenes. Many DNA viruses, such as human papillomavirus, inactivate tumor suppressor genes, but there is no convincing evidence that HBV or HCV can bind to p53 or RB proteins. Also, the HBV-encoded regulatory element, called HBx, disrupts normal growth of infected hepatocytes. Neither HBV nor HCV infects immune cells.

17 For the past 3 days, a 68-year-old woman has had a fever and cough productive of yellow sputum. On physical examination, there is dullness to percussion at the left lung base. A chest radiograph shows areas of consolidation in the left lower lobe. Despite antibiotic therapy, the course of the disease is complicated by abscess formation, and she dies. At autopsy, there is a bronchopleural fistula surrounded by a pronounced fibroblastic reaction. Small, yellow, 1- to 2-mm "sulfur granules" are grossly visible within the area of abscess formation. Which of the following organisms is most likely to produce these autopsy findings? □ (A) Actinomyces israelii □ (B) Blastomyces dermatitidis □ (C) Chlamydia pneumoniae □ (D) Klebsiella pneumoniae □ (E) Mycobacterium kansasii

1 7 (A) Actinomycetes that can produce chronic abscessing pneumonia, particularly in immunocompromised patients, include Actinomyces israelii and Nocardia asteroides. Sulfur granules, formed from masses of the branching, filamentous Robbins & Cotran Review of Pathology Pg. 145 organisms, are more likely to be seen in Actinomyces. Blastomyces dermatitidis infections tend to produce a granulomatous inflammatory process. Chlamydial infections produce an interstitial pattern similar to that of most viruses. Klebsiella infections, similar to other bacterial infections, can result in abscess formation, although without distinct sulfur granules. Mycobacterium kansasii infections are similar to M. tuberculosis infections in that granulomatous inflammation is prominent.

17 A 58-year-old woman has experienced an increasing feeling of fullness in the neck for the past 3 months, and she has noted a 3-kg weight loss during that time. On physical examination, there is a firm, fixed mass in a 3 × 5 cm area in the right side of the neck. CT scan shows a solid mass in the region of the right lobe of the thyroid gland. A biopsy of the mass is performed; the microscopic appearance of the specimen is shown in the figure. All areas of the tumor have similar morphology. Which of the following terms best describes this neoplasm? □ (A) Adenoma □ (B) Well-differentiated adenocarcinoma □ (C) Squamous cell carcinoma □ (D) Leiomyoma □ (E) Anaplastic carcinoma

1 7 (E) The cells shown in the figure show marked pleomorphism and hyperchromatism (anaplasia). A bizarre tripolar mitotic figure is present. This degree of anaplasia is consistent with a malignancy. An adenoma is a benign tumor of glandular origin. Adenocarcinomas and squamous cell carcinomas show differentiation into glandular or squamous tissues. Leiomyomas are benign mesenchymal tumors of smooth muscle origin.

19 An epidemiologic study analyzes health care benefits of cancer screening techniques applied to a population. Which of the following diagnostic screening techniques used in health care is most likely to have had the greatest impact on reduction in cancer deaths in developed nations? □ (A) Chest radiograph □ (B) Stool guaiac □ (C) Pap smear □ (D) Serum carcinoembryonic antigen assay □ (E) Urinalysis

1 9 (C) Because Pap smear screening can detect dysplasias and in situ carcinomas that can be treated before progression to invasive lesions, deaths from cervical carcinoma have steadily decreased since this screening method became widely available in the last half of the 20th century. A chest radiograph is an insensitive technique for detecting early lung cancers. Use of stool guaiac has had a minimal effect on rates of death from colorectal carcinomas, but physicians are cautioned not to indicate "rectal deferred" on the physical examination report, and hence contribute to the problem. Serum tumor markers have not proved useful as general screening techniques, although they are useful in selected circumstances. Urine cytology is better than urinalysis for detection of urothelial malignancies, but it does not have a high sensitivity.

10 A 60-year-old man has had persistent bloody diarrhea, abdominal cramps, and fever for the past week. On physical examination, his temperature is 38.1°C. He has mild diffuse abdominal pain. A stool sample is positive for occult blood. Sigmoidoscopic examination shows mucosal ulceration in the cecum and ascending colon. On colonoscopy, marked mucosal erythema with focal ulceration is seen from the rectum to the ascending colon. The ulcers do not penetrate the muscularis propria. A biopsy is performed, and the microscopic appearance of the specimen is shown in the figure. Which of the following infectious organisms is most likely to produce these findings? □ (A) Giardia lamblia □ (B) Entamoeba histolytica □ (C) Shigella flexneri □ (D) Salmonella enteritidis □ (E) Vibrio cholerae □ (F) Bacillus cereus

10 (B) Amebiasis is a common cause of dysentery in developing nations. The Entamoeba histolytica trophozoites can attach to colonic epithelium, invade, and lyse the epithelial cells. In some cases, there can be extensive mucosal involvement with characteristic flask-shaped (similar to an Erlenmeyer flask) ulcerations similar to those seen in other severe inflammatory bowel diseases. Giardiasis tends to involve the small intestine and produces variable inflammation, but no ulceration. Shigellosis can produce bloody dysentery with irregular superficial colonic mucosal ulceration, but the organisms typically do not invade beyond the lamina propria. Salmonellosis more typically involves the small intestine and in most cases produces self-limited enteritis, although more severe disease with dissemination to other organs can occur Robbins & Cotran Review of Pathology Pg. 144 with Salmonella typhi infection. Cholera is characterized by massive, secretory diarrhea without intestinal mucosal invasion or necrosis. Bacillus cereus is a cause of food poisoning (most often as a contaminant in reheated fried rice) and has a short incubation time.

10 A 50-year-old woman undergoes screening colonoscopy as part of a routine health maintenance workup. An isolated 1- cm pedunculated polyp is found in the sigmoid colon. The excised polyp histologically shows well-differentiated glands with no invasion of the stalk. Which of the following investigational research procedures can distinguish most clearly whether the polyp represents hyperplasia of the colonic mucosa or a tubular adenoma? □ (A) Histochemical staining for mucin □ (B) Flow cytometry to determine the frequency of cells in the S phase □ (C) Determination of clonality by pattern of X chromosome inactivation □ (D) Immunoperoxidase staining for keratin □ (E) Immunoperoxidase staining for factor VIII

10 (C) A true neoplasm is a monoclonal proliferation of cells, whereas a reactive proliferation of cells is not monoclonal. Reactive and neoplastic cellular proliferations may have similar histochemical and immunohistochemical staining patterns based on the type of cells that are present. Flow cytometry is effective at indicating the DNA content, aneuploidy, and growth fraction, but does not indicate clonality.

11 A 24-year-old woman has had bloody diarrhea for the past 4 days. On physical examination, she has a temperature of 38.3°C. The appearance of the rectum and descending colon on colonoscopy is shown in the figure. The patient is treated with antibiotics but develops a chronic arthritis after the diarrhea has resolved. HLA typing is done, and she is found to be HLA-B27 positive. Which of the following organisms is most likely to be identified in her diarrheal stool? □ (A) Vibrio cholerae □ (B) Shigella flexneri □ (C) Entamoeba histolytica □ (D) Salmonella typhi □ (E) Helicobacter pylori

11 (B) Shigella organisms elaborate a shiga toxin that damages colonic epithelial cells. The colonic mucosa is intensely inflamed, with ulcerations and pseudomembrane formation (pale, white patches). Shigella flexneri infections in individuals positive for HLA-B27 can lead to Reiter syndrome, with chronic arthritis. Vibrio organisms elaborate an exotoxin and do not invade and destroy intestinal epithelium. Entamoeba histolytica can produce bloody diarrhea, but not Reiter syndrome. Typhoid fever causes many systemic problems, but not arthritis. The presence of Helicobacter pylori in gastric mucus drives the processes of chronic gastritis and ulceration of gastric and duodenal mucosal surfaces.

11 A 66-year-old woman has worked all of her life on a small family farm on the Kanto plain near Tokyo. She has had no previous major illnesses, but has been feeling increasingly tired and weak for the past year. On physical examination, she is afebrile, but appears pale. Laboratory studies show hemoglobin, 11.3 g/dL; hematocrit, 33.8%; platelet count, 205,200/mm3; and WBC count, 64,000/mm3. Immunophenotyping yields the findings shown in the figure. Assuming that the dominant cell population is clonal, which of the following viral agents is most likely involved in this patient's disease process? □ (A) Human papillomavirus □ (B) HIV-1 □ (C) Epstein-Barr virus □ (D) Human T cell lymphotropic virus type 1 □ (E) Hepatitis B virus

11 (D) The largest cell population, determined to be clonal, is marking for CD4. This patient has a T cell leukemia, which develops in approximately 1% of individuals infected with human T cell lymphotropic virus type 1. Human papillomavirus is best known for causing squamous epithelial dysplasias and carcinomas. HIV-1 infection causes AIDS. Infection with Epstein-Barr virus is associated with various cancers, including Burkitt lymphoma and nasopharyngeal carcinoma. Infection with hepatitis B virus may result in hepatic cirrhosis, in which hepatocellular carcinoma may arise.

12 A 50-year-old woman saw her physician after noticing a mass in the right breast. Physical examination showed a 2-cm mass fixed to the underlying tissues beneath the areola and three firm, nontender, lymph nodes palpable in the right axilla. There was no family history of cancer. An excisional breast biopsy was performed, and microscopic examination showed a well-differentiated ductal carcinoma. Over the next 6 months, additional lymph nodes became enlarged, and CT scans showed nodules in the lung, liver, and brain. The patient died 9 months after diagnosis. Which of the following molecular abnormalities is most likely to be found in this setting? □ (A) Inactivation of one BRCA1 gene copy □ (B) Deletion of one p53 gene copy □ (C) Amplification of the ERBB2 (HER2) gene □ (D) Deletion of an RB gene locus □ (E) Fusion of BCR and C-ABL genes

12 (C) Increased expression of ERBB2 (HER2) can be detected immunohistochemically in the biopsy specimen. One third of breast cancers may show this change. Such amplification is associated with a poorer prognosis. Detection of a specific gene product in the tissue has value for determination of prognosis. BRCA1 and p53 mutations, if inherited in the germ line, can predispose the patient to breast cancer and other tumors. With BRCA1, there is family history of breast cancer, and p53 mutation predisposes to many types of cancers. An inherited deletion of RB gene predisposes to retinoblastoma. The BCR-ABL fusion product, seen in chronic myeloid leukemia, results from t(9;22).

12 A 23-year-old woman has had increasing delirium for 2 days and is admitted to the hospital. On physical examination, she has acute pharyngitis with an overlying dirty-white, tough mucosal membrane. Paresthesias with decreased vibratory sensation are present in the extremities. On auscultation, there is an irregular cardiac rhythm. A chest radiograph shows cardiomegaly. A Gram stain of the pharyngeal membrane shows numerous small, gram-positive rods in a fibrinopurulent exudate. Which of the following is the most likely mechanism for development of cardiac disease in this patient? □ (A) Microabscess formation □ (B) Endotoxin-mediated hypotension and shock □ (C) Vasculitis with thrombosis and infarction Robbins & Cotran Review of Pathology Pg. 128 □ (D) Granulomatous inflammation □ (E) Elaboration of exotoxin

12 (E) This woman has diphtheria. The Corynebacterium diphtheriae organisms proliferate in the inflammatory membrane that covers the pharynx and tonsils. These organisms elaborate an exotoxin that produces myocarditis and neuropathy. The organisms do not disseminate to cause inflammation or abscesses or vasculitis elsewhere in the body. Endotoxins tend to be elaborated by gram-negative bacterial organisms. Granulomatous inflammation is more typical of mycobacterial and fungal infections.

13 A 15-year-old boy has a small eschar on his left forearm around the site of a tick bite. A hemorrhagic rash involving the trunk, extremities, and palms and soles develops over the next few days. Small, 0.2- to 0.4-cm foci of skin necrosis develop on the fingers and toes. Which of the following organisms is most likely to produce these findings? □ (A) Rickettsia rickettsii □ (B) Mycobacterium leprae □ (C) Yersinia pestis □ (D) Borrelia burgdorferi □ (E) Leishmania braziliensis

13 (A) This patient has Rocky Mountain spotted fever, which occurs sporadically in the United States in areas other than the Rocky Mountains. Rickettsial diseases produce signs and symptoms from damage to vascular endothelium and smooth muscle similar to a vasculitis. Thrombosis of the affected blood vessels is responsible for foci of skin necrosis. Hansen's disease, produced by Mycobacterium leprae, results in skin anesthesia that predisposes to recurrent injury. Plague, caused by Yersinia pestis, can produce focal skin necrosis at the site of a flea bite. Lyme disease, caused by Borrelia burgdorferi, can produce an erythema chronicum migrans of skin at the site of a tick bite. Mucocutaneous leishmaniasis mainly involves the nasal and oral regions.

13 In a family of five children, a 12-year-old girl and a 14-year-old boy have been affected by skin nodules that have developed over the past 5 years. On physical examination, both children are of appropriate height and weight. The skin lesions are 1- to 3-cm maculopapular nodules that are erythematous to brown-colored and have areas of ulceration. Biopsy specimens of the skin lesions show squamous cell carcinoma. The children have no history of recurrent infections, and their parents and other relatives are unaffected. Which of the following mechanisms is most likely to produce neoplasia in these children? □ (A) Infection with human papillomavirus □ (B) Failure of nucleotide excision repair of DNA □ (C) Ingestion of food contaminated with Aspergillus flavus □ (D) Inactivation of p53 □ (E) Chromosomal translocation

13 (B) The children described in the question have an autosomal recessive condition known as xeroderma pigmentosum. Affected individuals have extreme photosensitivity, with a 2000-fold increase in the risk of skin cancers. The DNA damage is initiated by exposure to ultraviolet light; however, nucleotide excision repair cannot occur normally in xeroderma pigmentosum. Human papillomavirus is a sexually transmitted disease that is associated with the development of genital squamous cell carcinomas. Aspergillus flavus, found on moldy peanuts and other foods, produces the potent hepatic carcinogen aflatoxin B1. Inactivation of the p53 tumor suppressor gene is found in many sporadic human cancers and in some familial cancers, but these cancers are not limited to the skin. Chromosomal translocations are often involved in the development of hematologic malignancies, although they are not often seen in skin cancers.

14 A 55-year-old man visits the physician because of hemoptysis and worsening cough. On physical examination, wheezes are auscultated over the right lung posteriorly. A chest radiograph shows a 6-cm perihilar mass on the right. A fine-needle aspiration biopsy yields cells consistent with non-small cell bronchogenic carcinoma. Molecular analysis of the neoplastic cells shows a p53 gene mutation. Which of the following mechanisms has most likely produced the neoplastic transformation? □ (A) Inability to hydrolyze GTP □ (B) Microsatellite instability □ (C) Lack of necrosis □ (D) Loss of cell cycle arrest □ (E) Transcriptional activation

14 (D) The p53 mutation involving both alleles is one of the most common genetic mutations in human cancers, including the most common—lung, colon, and breast cancers. The loss of this tumor suppressor indicates that the cell cycle is not properly arrested in the late G1 phase; when DNA damage occurs, DNA repair cannot be completed before the cell proliferates. Inability to hydrolyze GTP is a result of RAS oncogene activation. Microsatellite instability occurs with mutation in genes, such as hMSH2, that repair DNA damage. BCL2 mutation is one of the best-known mechanisms for apoptotic arrest in neoplasms. Transcriptional activation is a feature of the MYC proto-oncogene.

14 A 40-year-old woman has had a high fever and swelling, warmth, and tenderness of the right arm for the past 3 days. On physical examination, she has a temperature of 39.4°C and the appearance shown in the figure. She receives antibiotic therapy and recovers. Which of the following organisms is most likely to produce these findings? □ (A) Clostridium botulinum □ (B) Escherichia coli □ (C) Neisseria gonorrhoeae □ (D) Staphylococcus epidermidis □ (E) Streptococcus pyogenes

14 (E) The rash and edema are manifestations of streptococcal erysipelas. Erysipelas is usually caused by group A or group C streptococci. Streptolysins elaborated by these organisms aid in the spread of the infection. Clostridium botulinum elaborates an exotoxin that, when ingested, results in paralysis. Escherichia coli produces various infections, but skin infections are uncommon. Neisseria gonorrhoeae is best known as a sexually transmitted disease, and a rash is possible, although usually there is no pronounced swelling. Staphylococcus epidermidis is usually considered a contaminant in cultures.

15 In an epidemiologic study of individuals who died in a worldwide pandemic after World War I, many individuals were shown to have contracted an influenza pneumonia. At the beginning of the 21st century, a similar epidemic is still possible from a virulent form of influenza for which no vaccine is available. Molecular analysis of samples of tissues showed changes in the virus responsible for these virulent forms of influenza. Which of the following changes most likely occurred in the virus to increase its virulence? □ (A) Mutations in DNA encoding envelope proteins □ (B) Increased binding to intercellular adhesion molecule-1 (ICAM-1) receptor □ (C) Ability to elaborate exotoxins □ (D) Recombination with RNA segments from animal viruses □ (E) Acquisition of antibiotic resistance genes

15 (D) The influenza pandemic in 1918 resulted from an antigenic shift in the influenza A type. This antigenic shift occurs when there is recombination with RNA sequences of influenza viruses found in animals such as pigs ("swine flu") or birds ("avian flu"). A swine flu virus has been identified as a cause of the 1918 pandemic. The H5N1 strain of influenza has been found in bird populations in modern times. Mutations in the viral hemagglutinin and neuraminidase envelope genes are responsible for epidemics. These mutations allow evasion from host antibodies. Influenza viruses do not bind to intercellular adhesion molecule-1 (ICAM-1) receptors; rhinoviruses do. Viruses do not make exotoxins and do not acquire antibiotic resistance.

15 A 5-year-old child has difficulty with vision in the right eye. On physical examination, there is leukokoria of the right eye, consistent with a mass in the posterior chamber. MR imaging shows a mass that nearly fills the globe. The child undergoes enucleation of the right eye. Molecular analysis of the neoplastic cells indicates absence of both copies of a tumor suppressor gene that controls the transition from the G1 to the S phase of the cell cycle. Which of the following genes is most likely to have the mechanism of action that produced this neoplasm? □ (A) BCR-ABL □ (B) BCL2 □ (C) hMSH2 □ (D) K-RAS □ (E) NF1 □ (F) p53 □ (G) RB

15 (G) The RB gene is a classic example of the two-hit mechanism for loss of tumor suppression. About 60% of these tumors are sporadic. Others are familial, and there is inheritance of a mutated copy of the RB gene. Loss of the second copy in retinoblasts leads to the occurrence of retinoblastoma in childhood. Researchers do not know why patients who inherit a mutant RB gene through the germline develop retinoblastoma rather than other types of tumors. The RB gene controls the G1 to S transition of the cell cycle; with loss of both copies, this important checkpoint in the cell cycle is lost. The BCR-ABL fusion gene in chronic myelogenous leukemia is an example of overexpression of a gene product producing neoplasia. The BCL2 gene is an inhibitor of apoptosis. The hMSH2 gene is present in most cases of hereditary nonpolyposis colon cancer and functions in DNA repair. Many cancers have the K-RAS gene, which acts as an oncogene. The NF1 gene product acts as a tumor suppressor; this is a component of neurofibromatosis (which usually does not involve the eye), and the neoplasms typically appear at a later age. Many cancers have the p53 tumor suppressor gene mutation, but this is not typical of childhood ocular neoplasms.

16 A 52-year-old man has a fever and cough that worsens over several days. On physical examination, his temperature is 38.2°C. On auscultation of the chest, diffuse crackles are heard at the right lung base. Laboratory studies show hemoglobin, 13.3 g/dL, hematocrit, 40%; platelet count, 291,800/mm3; and WBC count, 13,240/mm3 with 71% segmented neutrophils, 7% bands, 16% lymphocytes, and 6% monocytes. Klebsiella pneumoniae is cultured from the patient's Robbins & Cotran Review of Pathology Pg. 129 sputum. His condition improves after a course of antibiotic therapy with gentamicin. Which of the following complications of this infection is the patient most likely to develop? □ (A) Gas gangrene □ (B) Cavitary granulomas □ (C) Abscess formation □ (D) Bullous emphysema □ (E) Adenocarcinoma

16 (C) Bacterial infections are marked by suppurative inflammation, and a virulent organism such as Klebsiella can lead to tissue destruction with abscess formation. Gas-forming bacteria, such as Clostridium organisms, are unusual as a cause of respiratory infections. Granulomatous inflammation is characteristic of mycobacterial or fungal infections. Infections of the lung do not result in emphysema. Carcinomas are not sequelae of bacterial infections.

16 A 60-year-old man comes to his physician because he has noted a mass in his neck that has increased rapidly in size over the past 2 months. On physical examination, a firm, nontender, 10-cm mass in the left lateral posterior neck that appears to be fused to cervical lymph nodes is palpated. Hepatosplenomegaly is noted. A head CT scan reveals a mass in the Waldeyer ring near the pharynx. A biopsy of the neck mass is performed, and on microscopic examination the biopsy specimen shows abnormal lymphoid cells with many mitotic figures and many apoptotic nuclei. The patient is treated with a cocktail of cell cycle-acting chemotherapeutic agents. The cervical and oral masses shrink dramatically over the next month. Based on his history and response to treatment, the tumor cells are most likely to have which of the following features? □ (A) Limited capacity to metastasize □ (B) Polyclonality □ (C) Poor vascularity □ (D) High growth fraction □ (E) Strong expression of tumor antigens

16 (D) Some neoplasms, including certain lymphomas, have a high proportion of cells in the replicative pool (i.e., have high growth fraction). They grow rapidly and respond rapidly to drugs that kill dividing cells. Monoclonality rather than polyclonality is typical of malignant tumors. Similarly, poor vascularity would not favor rapid growth. Tumors that are highly antigenic are likely to be controlled by the immune system and not to be rapidly growing.

18 A 59-year-old man has recently noticed blood in his urine. Cystoscopy shows a 4-cm exophytic mass involving the right bladder mucosa near the trigone. Biopsy specimens are obtained, and the patient undergoes a radical cystectomy. Examination of the excised specimen shows that a grade IV urothelial cell carcinoma has infiltrated the bladder wall. Which of the following statements regarding these findings is most appropriate? □ (A) The neoplasm is a metastasis □ (B) The patient has a poorly differentiated neoplasm □ (C) A paraneoplastic syndrome is likely □ (D) The stage of the neoplasm is low □ (E) The patient is probably cured of the cancer

18 (B) Cancer grading systems are typically denoted by I to III or I to IV, and increase with worse differentiation (more anaplasia). A transitional cell carcinoma would be expected at this site. Bladder cancers are not commonly associated with paraneoplastic syndromes. Infiltration through the wall indicates a high stage. The cure rate for this high-grade, high-stage cancer is poor. Determination of the presence of metastases is part of staging, not grading.

18 An 11-year-old boy had episodic fevers for 1 week and then developed a severe headache. He became progressively more somnolent and died 1 week later. At autopsy, there was marked diffuse cerebral edema with areas of cerebral softening. The microscopic appearance of a cerebral vein is shown. Which of the following organs is most likely to serve as the reservoir for proliferation of the infectious agent producing this disease? □ (A) Heart □ (B) Liver □ (C) Brain □ (D) Lymph nodes □ (E) Spleen

18 (B) This boy had malaria. After the infective mosquito bite, Plasmodium falciparum sporozoites invade liver cells and reproduce asexually. When the hepatocytes rupture, they release thousands of merozoites that infect RBCs. The infected RBCs circulate and can bind to endothelium in the brain. Small cerebral vessels become plugged with the RBCs, resulting in ischemia. The other listed options also could be secondarily involved by vascular thromboses.

19 A 29-year-old man has had hematuria for the past month. On physical examination, he is afebrile. There is diffuse lower abdominal tenderness, but no palpable masses. An abdominal radiograph shows a small bladder outlined by a rim of calcification. Cystoscopy is performed, and the entire bladder mucosa is erythematous and granular. Biopsy samples are taken. Which of the following histologic findings is most likely to be seen in these samples? □ (A) Eggs of Schistosoma haematobium □ (B) Larvae of Trichinella spiralis □ (C) Taenia solium cysts □ (D) Acid-fast bacilli □ (E) Migrating Ascaris lumbricoides

19 (A) Schistosoma hematobium is seen in Africa, particularly the Nile Valley, in areas where irrigation has expanded the range of the host snails. It infects the wall of the urinary bladder, causing severe granulomatous inflammation, fibrosis, and calcification. Trichinella spiralis infects striated muscle. Cysticercosis can have a wide tissue distribution, but the brain is most often affected. Mycobacterial infections of the urinary tract are uncommon and do not cause bladder fibrosis. Ascariasis involves the lower gastrointestinal tract, and the worms reside in the lumen.

2 A 48-year-old woman notices a lump in her left breast. On physical examination, the physician palpates a firm, nonmovable, 2-cm mass in the upper outer quadrant of the left breast. There are enlarged, firm, nontender lymph nodes in the left axilla. A fine-needle aspiration biopsy is performed, and the cells present are consistent with carcinoma. A mastectomy with axillary lymph node dissection is performed, and carcinoma is present in two of eight axillary nodes. Which of the following factors is most likely responsible for the lymph node metastases? □ (A) Increased laminin receptors on tumor cells □ (B) Presence of keratin in tumor cells □ (C) Diminished apoptosis of tumor cells □ (D) Tumor cell monoclonality □ (E) Lymphadenitis

2 (A) Several pathologic mechanisms play a role in the development of tumor metastases. The tumor cells first must become discohesive and detach from the primary site and then attach elsewhere to become metastases. Tumor cells tend to have many more laminin receptors than do normal cells, allowing them to attach more readily to basement membranes at distant sites. Keratin is a marker of epithelial differentiation, not metastatic ability. A reduction in apoptosis allows greater proliferation, but not metastases. Monoclonality is a feature of neoplasia, but further tumor heterogeneity helps to increase the chance for metastases to occur. Inflammation probably does not play a major role in metastasis.

2 A 33-year-old woman develops fever with right upper abdominal pain 6 weeks after a trip to Central America. During the last week of her trip, she had blood-tinged watery diarrhea, but it subsided shortly after her return home. On physical examination, her temperature is 37.6°C. There is moderate tenderness on palpation of the right upper quadrant, and the liver span is increased. Laboratory studies show a total bilirubin level of 5.4 mg/dL, a direct bilirubin concentration of 4.9 mg/dL, and alkaline phosphatase level of 175 U/L. An abdominal CT scan shows a 7-cm right hepatic mass with central necrosis and discrete borders. Which of the following organisms is most likely to produce these findings? □ (A) Giardia lamblia □ (B) Salmonella typhi □ (C) Entamoeba histolytica □ (D) Campylobacter jejuni □ (E) Yersinia enterocolitica □ (F) Staphylococcus aureus

2 (C) Amebic liver abscess is an uncommon complication of amebiasis. The colonic lesions typically have disappeared by the time the liver lesions appear. Entamoeba histolytica organisms can invade the colonic submucosa, gaining access to venules draining to the portal system. Giardiasis is caused by an intestinal parasite and produces watery diarrhea. Typhoid fever is a systemic disease that produces splenomegaly more so than hepatomegaly, and abscesses are uncommon. Campylobacter and Yersinia can produce various presentations of diarrhea, but abscesses would not be expected. Staphylococcal enterotoxin typically produces abdominal pain and diarrhea within hours of ingestion.

26 A 51-year-old man who works in a factory that produces plastic pipe has experienced weight loss, nausea, and vomiting over the past 4 months. On physical examination, he has tenderness to palpation in the right upper quadrant of the abdomen, and the liver span is increased. Laboratory findings include serum alkaline phosphatase, 405 U/L; AST, 45 U/L; ALT, 30 U/L; and total bilirubin, 0.9 mg/dL. An abdominal CT scan shows a 12-cm mass in the right lobe of the liver. A liver biopsy is performed, and microscopic examination shows an angiosarcoma. The patient has most likely been exposed to which of the following agents? □ (A) Arsenic □ (B) Asbestos □ (C) Benzene □ (D) Beryllium □ (E) Nickel □ (F) Vinyl chloride □ (G) Naphthalene

26 (F) Vinyl chloride is a rare cause of liver cancer. This causal relationship was easy to show, however, because hepatic angiosarcoma is a rare neoplasm. Arsenic is a risk factor for skin cancer. Asbestos exposure is linked to pleural malignant mesothelioma and to bronchogenic carcinomas in smokers. Benzene exposure is linked to leukemias. Beryllium exposure can produce interstitial lung disease and lung cancer. Nickel exposure increases the risk of respiratory tract cancers. Exposure to naphthalene compounds is a risk factor for cancer of the urinary tract.

24 At a convention of veterans, several of the participants begin to develop respiratory difficulty and fever. The affected men are between 58 and 73 years old, they are all smokers, and many have chronic obstructive lung disease. On physical examination, the men have temperatures ranging from 37.3°C to 38.4°C. On auscultation, crackles are heard in the lung bases. Chest radiographs show extensive pulmonary infiltrates with small abscesses. Sputum specimens show as many macrophages as neutrophils on the cytologic smears. Which of the following organisms is most likely to be identified in the sputum samples of these patients? □ (A) Cytomegalovirus □ (B) Pneumocystis carinii □ (C) Legionella pneumophila □ (D) Burkholderia cepacia □ (E) Listeria monocytogenes

2 4 (C) The original outbreak for which this disease was named occurred at an American Legion convention in Philadelphia. Legionella pneumophila is a facultative parasite of macrophages. A high ratio of macrophages to neutrophils is characteristic of the infection. Individuals with immunosuppression are at risk for cytomegalovirus and Pneumocystis pneumonia. Burkholderia cepacia is most often seen in patients with cystic fibrosis who have extensive bronchiectasis. Listeria monocytogenes can produce disseminated disease with meningitis in immunocompromised adults.

25 A 25-year-old man has noted worsening pain on the right side of his face for the past 24 hours. On examination, there is marked tenderness and swelling inferior to the zygomatic arch and lateral to the nasolabial fold on the right. Laboratory studies show hemoglobin, 14.6 g/dL; WBC count, 9900/mm3; serum creatinine, 2 mg/dL; sodium, 151 mmol/L; potassium, 5.4 mmol/L; chloride, 119 mmol/L; bicarbonate, 8 mmol/L; and glucose, 483 mg/dL. A head CT scan shows soft tissue swelling and bony destruction around the right maxillary sinus. A biopsy is performed; the figure shows the findings on microscopic examination. Which of the following organisms is the most likely causative agent for this patient's infection? □ (A) Aspergillus niger □ (B) Actinomyces israelii □ (C) Candida albicans □ (D) Clostridium perfringens □ (E) Cryptococcus neoformans □ (F) Mucor circinelloides

2 5 (F) This patient is in diabetic ketoacidosis, which is a significant risk factor for mucormycosis. Note the broad, nonseptated hyphae. In contrast, Aspergillus organisms have thinner hyphae with acute angle branching and septations. Actinomyces organisms are long, filamentous gram-positive bacilli. Candida infections are typically superficial, and a Gram stain shows gram-positive budding cells with pseudohyphae. Large, gram-positive rods are characteristic of Clostridium perfringens, which can contaminate open wounds and produce gas gangrene.

26 A 28-year-old woman from Venezuela with a history of rheumatoid arthritis develops painful swelling of her hands and feet. She is treated with corticosteroid therapy. A month later, she develops profuse, watery diarrhea along with fever and cough. On examination, she has a temperature of 37.3°C. Laboratory studies show hemoglobin, 13.5 g/dL; WBC count, 12,900/mm3; and platelet count, 242,000/mm3. The WBC differential count shows 59 segmented neutrophils, 5 bands, 16 lymphocytes, 8 monocytes, and 12 eosinophils. Microscopic examination of a stool specimen shows ova and small rhabditoid larvae. Similar larvae are present in a sputum specimen. Which of the following infectious diseases is most Robbins & Cotran Review of Pathology Pg. 132 likely to produce these findings? □ (A) Cysticercosis □ (B) Onchocerciasis □ (C) Schistosomiasis □ (D) Strongyloidiasis □ (E) Trichinosis

2 6 (D) The rhabditoid larvae of Strongyloides stercoralis can become invasive filariform from autoinfection in immunocompromised hosts, so-called "hyperinfection" with involvement of multiple organs. Immunocompetent hosts typically have only diarrhea. Parasites, particularly worms, crawling through tissues incite a marked eosinophilia. Cysticercosis from eating uncooked pork can result in the release of larvae that penetrate the gut wall and disseminate hematogenously, often settling in gray and white cerebral tissue, where they develop into cysts. Onchocerciasis occurs as a result of infection with the filarial nematode Onchocerca volvulus and leads to formation of a subcutaneous nodule. Schistosoma mansoni or Schistosoma japonicum infections have adult female worms in the portal venous system that release eggs that can produce hepatic fibrosis; Schistosoma haematobium worms live in veins near the bladder and release eggs that result in hematuria. Eating infected meat, typically uncooked pork, can lead to trichinosis; Trichinella encysts in striated muscle to produce fever and myalgias.

27 A 39-year-old woman underwent a routine health maintenance examination for the first time in many years. A Pap smear was obtained, and the result reported was abnormal. On pelvic examination, a red, slightly raised, 1-cm lesion on the anterior ectocervix at the 2-o'clock position was excised and biopsied. The microscopic appearance on medium-power magnification is shown in the figure. Which of the following statements best characterizes the patient's condition? Robbins & Cotran Review of Pathology Pg. 106 □ (A) A primary site should be sought □ (B) This is a high-grade lesion □ (C) The cell of origin is a fibroblast □ (D) A chest radiograph would show nodules □ (E) Local excision would be curative

2 7 (E) The figure shows an in situ carcinoma of the squamous cervical epithelium with neoplastic growth above the basement membrane. Such cancers, limited to the epithelium, are noninvasive, and local excision has a 100% cure rate. In situ lesions do not give rise to metastases. Lesions limited to the epithelium are low grade. Because it originated in the epithelium, this neoplasm is not derived from fibroblasts.

20 During a routine health maintenance examination, a 46-year-old man is found to have an enlarged, nontender supraclavicular lymph node that is palpable on physical examination. The 2-cm node is excised. Histologically, the nodal architecture is effaced by a monomorphous population of small lymphocytes. Which of the following procedures would best confirm that the patient has a malignancy? □ (A) Peripheral WBC count and differential cell count □ (B) Flow cytometry of nodal tissue for DNA content □ (C) Electron microscopy to determine cellular ultrastructure □ (D) Southern blot analysis to show monoclonality □ (E) Determination of the serum lactate dehydrogenase level

20 (D) Monoclonality is the hallmark of a malignancy. In the diagnosis of a leukemia, the WBC count is helpful, but not definitive. The DNA content analysis alone cannot define a malignancy; Southern blot analysis for T-cell or B-cell receptor gene rearrangements can define monoclonality. Electron microscopy is an adjunct to diagnosis of the type of tumor. Robbins & Cotran Review of Pathology Pg. 118 Lactate dehydrogenase levels are often increased with lymphoid proliferations, but are not diagnostic of the type of proliferation.

20 A 24-year-old college student comes to the health service because he has had a cough, fever, and shortness of breath for the past 3 weeks when walking. The results of cardiac examination are normal, but crackles are heard in both lungs. A chest radiograph shows patchy infiltrates in the lungs. Laboratory studies show an elevated cold agglutinin titer. A presumptive clinical diagnosis of Mycoplasma pneumoniae is made, and the patient responds to erythromycin therapy. Which of the following histologic changes is most likely responsible for the pulmonary symptoms in this patient? □ (A) Neutrophils within bronchioles, extending into alveoli □ (B) Granulomas with Langhans giant cells □ (C) Pulmonary infarcts with vascular occlusion by microorganisms □ (D) Mononuclear interstitial infiltrate □ (E) Collection of neutrophils and fibrin in the pleural space

20 (D) Mycoplasma infections lead to a primary atypical pneumonia in which there are no alveolar infiltrates, but there is prominent interstitial inflammation with lymphocytes, histiocytes, and plasma cells. Alveolar and bronchiolar neutrophilic exudates suggest a bacterial agent causing pneumonia. Granulomas with Langhans-type giant cells are typical of tuberculosis. Proliferation of microorganisms with vascular occlusion and infarction is most typical of Aspergillus fungal infections. An empyema with neutrophils suggests a bacterial cause for pneumonia with spread to pleura.

21 An epidemiologic study investigates the potential cellular molecular alterations that may contribute to the development of cancers in a population. Data analyzed from resected colonic lesions show that changes are occurring that show the evolution of a sporadic colonic adenoma into an invasive carcinoma. Which of the following best describes the mechanism producing these changes? □ (A) Activation of proto-oncogenes by chromosomal translocation □ (B) Stepwise accumulation of multiple proto-oncogene and tumor suppressor gene mutations □ (C) Extensive regeneration of tissues increasing the mutation rate in regenerating cells □ (D) Inheritance of defects in DNA repair genes that increase the susceptibility to develop cancer □ (E) Overexpression of growth factor receptor genes

21 (B) Development of colonic adenocarcinoma typically takes years, during which time numerous mutations occur within the mucosa, including mutations involving such genes as APC (adenomatous polyposis coli), K-RAS, and p53. The accumulation of mutations, rather than their occurrence in a specific order, is most important in the development of a carcinoma. Activation of proto-oncogenes, extensive regeneration, faulty DNA repair genes, and amplification of growth factor receptor genes all contribute to the development of malignancies, but they are not sufficient by themselves to produce a carcinoma from an adenoma of the colon.

21 The "fun ship" leaves port from Miami on a cruise to the Caribbean. On the third day out of port, 10 adult passengers experience sudden onset of nausea and abdominal cramps, followed by periods of watery diarrhea. Some of the affected passengers also experience vomiting, and most have headaches, myalgias, or abdominal pain. On physical examination, five patients have a temperature of 38°C, but there are no other abnormal findings. Stool smears show no RBCs, leukocytes, ova, or cysts. The affected passengers recover without therapy over the next 3 days. Which of the following is the most likely causative agent? □ (A) Giardia lamblia □ (B) Norwalk-like virus □ (C) Rotavirus □ (D) Shigella dysenteriae □ (E) Yersinia pseudotuberculosis □ (F) Vibrio parahaemolyticus

21 (B) Norwalk-like viruses are members of the Caliciviridae family of viruses (also known as the enteric caliciviruses) and are transmitted by hands contaminated through the fecal-oral route, directly from person to person, through contaminated food or water, or by contact with contaminated surfaces (known as fomites). They are the leading cause of outbreaks of gastroenteritis. Infections occur year-round, with a clear peak in the winter. More than 80% of adults in developed and developing countries have antibodies to Norwalk-like viruses. Giardiasis usually produces self-limited, watery diarrhea and is acquired from untreated water supplies. Rotavirus is more common in children. Shigellosis typically leads to a more severe, sometimes bloody, diarrhea. Yersinia pseudotuberculosis and Y. enterocolitica can produce a syndrome of mesenteric adenitis and terminal ileitis with abdominal pain that may mimic acute appendicitis. Vibrio parahaemolyticus causes an acute gastroenteritis with explosive watery diarrhea, vomiting, nausea, abdominal cramps, and headaches.

22 A 49-year-old man has a lump near his right shoulder that has been increasing in size for the past 8 months. On physical examination, the physician palpates a 4-cm, firm, nontender mass in the right supraclavicular region. The mass is excised, and microscopically it is diagnosed as follicular lymphoma. Karyotypic analysis of the cells shows a chromosomal translocation, t(14;18), involving the immunoglobulin heavy-chain gene. Which of the following genes is most likely to have undergone mutation to produce these findings? □ (A) APC (tumor suppressor gene) □ (B) BCL1 (cyclin gene) □ (C) BCL2 (anti-apoptosis gene) □ (D) BRCA1 (DNA repair gene) □ (E) C-MYC (transcription factor gene) □ (F) ERBB2 (growth factor receptor gene) □ (G) IL2 (growth factor gene) □ (H) K-RAS (GTP-binding protein gene) □ (I) p53 (DNA damage response gene)

22 (C) This is an example of chromosomal translocation that brings BCL2, an anti-apoptosis gene, close to another gene (immunoglobulin heavy-chain gene). The BCL2 gene becomes subject to continuous stimulation by the adjacent enhancer element of the immunoglobulin gene, leading to overexpression. The APC gene is mutated in sporadic colon cancers and cancers associated with familial polyposis coli. The BCL1 gene is mutated in mantle zone lymphoma, with t(11;14) that brings the cyclin gene on chromosome 11 to the immunoglobulin enhancer gene on chromosome 14. The BRCA1 and ERBB2 gene mutations are seen in some breast cancers. The IL2 mutation may be present in some T cell neoplasms. KRAS and p53 mutations are present in many cancers, but not typically lymphoid malignancies.

27 A 40-year-old man has had progressive enlargement of the left leg for the past 6 years, leading to the appearance shown in the figure. On physical examination, he is afebrile. He has inguinal lymphadenopathy and scrotal edema. Infection with which of the following organisms is most likely to be present? □ (A) Schistosoma mansoni □ (B) Echinococcus granulosis □ (C) Trichinella spiralis □ (D) Leishmania tropica □ (E) Wuchereria bancrofti

27 (E) This patient has elephantiasis, which results from lymphatic obstruction in the presence of an inflammatory reaction to the adult filarial worms Wuchereria bancrofti. Schistosomiasis may affect the liver or bladder most severely. Echinococcus produces hydatid disease of the liver, lungs, or bone. Trichinella encysts in striated muscle. Leishmania can involve the skin, causing ulceration, and can enlarge parenchymal organs.

22 A 44-year-old woman notices an erythematous papule on her left lower leg that becomes a ringlike rash and then subsides over several weeks. Over the next 5 months, she has migratory joint and muscle pain, substernal chest pain, and an irregular heart rhythm. The problems subside, but 2 years after the initial rash appeared, she develops a chronic arthritis involving the hips, knees, and shoulders. Which of the following is the most likely diagnosis? □ (A) Chagas disease □ (B) Dengue fever □ (C) Leishmaniasis □ (D) Leprosy □ (E) Lyme disease □ (F) Malaria □ (G) Syphilis

22 (E) The acute stage of Lyme disease is marked by the appearance of erythema chronicum migrans of the skin. As the Borrelia burgdorferi organisms proliferate and disseminate, systemic manifestations of carditis, meningitis, and migratory arthralgias and myalgias appear. These are followed 2 to 3 years after initial infection by arthritis involving the large joints. Chagas disease may be associated with acute and chronic myocarditis leading to heart failure; some patients have esophageal involvement, but arthritis and rash are not features of the disease. Hemorrhagic fever, or Dengue fever, caused by an arbovirus, can produce myositis and bone marrow suppression. Mucocutaneous ulcers may be seen with Leishmania braziliensis. Leprosy, or Hansen's disease, is associated with skin anesthesia and granuloma formation with nodular deformities of the skin. Malaria is not typically associated with skin lesions. In primary syphilis, a hard chancre may be present at the site of inoculation (usually the external genitalia); in secondary syphilis, a maculopapular rash may be present.

23 While repairing a fence on his farm, a 40-year-old man cuts the skin over his shin. The wound heals without any complications. Four days later, he develops muscle spasms of the face and extremities. These spasms worsen to the point of severe contractions. Which of the following actions by a toxin is responsible for the clinical features in this case? □ (A) Degradation of muscle cell membranes by phospholipase C □ (B) Inhibition of acetylcholine release at neuromuscular junctions □ (C) Stimulation of adenylate cyclase production □ (D) Cleavage of synaptobrevin in synaptic vesicles of neurons □ (E) Release of cytokine by T lymphocytes

23 (D) This man has tetanus. The contamination of a wound with Clostridium tetani can result in the elaboration of a potent neurotoxin. This toxin is a protease that cleaves synaptobrevin, a major transmembrane protein of the synaptic vesicles of the inhibitory neurons. Clostridium perfringens elaborates a variety of toxins, one of which (tetanospasmin) is a phospholipase. Inhibition of acetylcholine release is not a feature of infection. Cholera is produced when the toxin Robbins & Cotran Review of Pathology Pg. 146 elaborated by Vibrio cholerae stimulates epithelial cell adenylate cyclase. The toxin of Staphylococcus aureus is an enterotoxin that acts as a superantigen and stimulates T cell cytokine release.

23 A Pap smear obtained from a 29-year-old woman during a routine health maintenance examination is abnormal. She is currently asymptomatic, but has a history of multiple sexual partners. Cervical biopsy specimens are obtained. The representative microscopic appearance of a specimen is shown in the figure. Which of the following is the most likely diagnosis? □ (A) Adenocarcinoma □ (B) Carcinoma in situ □ (C) Dysplasia □ (D) Hamartoma □ (E) Melanoma □ (F) Mesothelioma □ (G) Small-cell anaplastic carcinoma □ (H) Squamous cell carcinoma

23 (H) Notice on the figure that the disorderly, atypical epithelial cells involve the entire thickness of the epithelium and extend through the underlying basement membrane, a process known as invasion. The ectocervix and the squamous metaplasia of endocervix give rise to squamous cell carcinoma. Carcinoma in situ is confined to the epithelium; if the basement membrane is breached, the lesion is no longer in situ, but rather invasive. An adenocarcinoma is a malignant neoplasm arising from glandular epithelium, such as the endocervix or endometrium, not the ectocervix. A dysplastic process could precede development of carcinoma in situ and squamous carcinoma; dysplasia involves only part of the thickness of the epithelium. A hamartoma contains a mixture of cell types common to a tissue site. Melanomas are malignant and tend to enlarge quickly; many are darkly pigmented. The benign counterpart to the melanoma is the nevus, which is quite common; nevi are usually light brown. A mesothelioma arises from the mesodermally derived lining of thoracic and abdominal body c

24 An epidemiologic study of cancer deaths recorded in the last half of the 20th century is conducted. The number of deaths for one particular type of cancer has been decreasing in developed nations, despite the absence of widespread screening programs. Which of the following neoplasms was most likely to be identified by this study? □ (A) Angiosarcoma of the liver □ (B) Gastric adenocarcinoma □ (C) Glioma of the brain □ (D) Leukemia □ (E) Lymphoma of the lymph nodes □ (F) Pancreatic adenocarcinoma

24 (B) The decrease in the number of gastric cancers may be related to reduced numbers of dietary carcinogens or a decrease in the prevalence of Helicobacter pylori infection; however, the exact reason is obscure. Angiosarcomas of the liver are quite rare; they are epidemiologically linked to vinyl chloride exposure. Cerebral gliomas are not as common as carcinomas; an urban legend links them to cell phone use, but legitimate epidemiologic studies have not made this link. Leukemias and lymphomas are not as common as carcinomas. Pancreatic adenocarcinoma is the sixth most common cause of cancer deaths in men and women, and the death rate is typically double the incidence because the prognosis for pancreatic cancer is so poor.

25 A 26-year-old woman has a lump in the left breast. On physical examination, the physician finds an irregular, firm, 2-cm mass in the upper inner quadrant of the breast. No axillary adenopathy is noted. A fine-needle aspirate of the mass shows carcinoma. The patient's 30-year-old sister was recently diagnosed with ovarian cancer, and 3 years ago her maternal aunt was diagnosed with ductal carcinoma of the breast and had a mastectomy. Which of the following genes is most likely to have undergone mutation to produce these findings? □ (A) BCL2 (anti-apoptosis gene) □ (B) BRCA1 (DNA repair gene) □ (C) EGF (epidermal growth factor gene) □ (D) ERBB2 (growth factor receptor gene) □ (E) HST1 (fibroblast growth factor gene) □ (F) IL2 (growth factor gene) □ (G) K-RAS (GTP-binding protein gene) □ (H) Lyn (tyrosine kinase gene)

25 (B) Approximately 5% to 10% of breast cancers are familial, and 80% of these cases result from mutations in the BRCA1 and BRCA2 genes. Onset of these familial cancers occurs earlier in life than the sporadic cancers. The protein products of these genes are involved in DNA repair. BCL2 is overexpressed in some lymphoid neoplasms. ERBB2 overexpression is present in some sporadic breast cancers; other EGF alterations can be seen in lung, bladder, gastrointestinal, ovarian, and brain neoplasms. The HST1 mutation is seen in some gastric cancers. IL2 overexpression is associated with some T cell neoplasms. K-RAS overexpression is seen in many cancers, including some breast cancers, but the early age of onset and family history in this case strongly suggest BRCA mutations. The Lyn mutation is seen in some immunodeficiency states.

28 A 22-year-old man develops a rubbery, red, 1-cm chancre on his right forearm. Three months later, he develops splenomegaly and lymphadenopathy. Two months later, he dies as a result of progressive wasting with cachexia and decreased mentation. At the time of his death, a peripheral blood smear had the appearance shown in the figure. Where is this disease most likely to have been acquired? □ (A) West Africa □ (B) Central America □ (C) Southeast Asia □ (D) Southern Europe □ (E) Polynesia

28 (A) The findings are consistent with African trypanosomiasis, or sleeping sickness. The eradication of the tsetse fly vector has been a priority for decades. Filarial worms endemic in parts of Central America, Southeast Asia, and Polynesia also can appear in blood, but are smaller in size and do not lead to chronic wasting. Filariasis is not endemic in Europe.

28 A 61-year-old woman has felt a lump in her breast for the past 2 months. On physical examination, there is a firm, 2-cm mass in the right breast. An excisional biopsy specimen of the mass shows carcinoma. Immunoperoxidase stains for protease cathepsin D and matrix metalloproteinase-9 are performed on the microscopic tissue section and show pronounced cytoplasmic staining in the tumor cells. Which of the following characteristics is most likely to be predicted by this marker? □ (A) Angiogenesis □ (B) Invasiveness □ (C) Differentiation □ (D) Heterogeneity □ (E) Aneuploidy

28 (B) The elaboration of various enzymes by tumor cells aids in degradation of extracellular matrix and invasiveness. Cathepsin D is a cysteine proteinase that cleaves various substrates, such as fibronectin and laminin. High levels of this enzyme in tumor cells are associated with greater invasiveness. Angiogenesis can be mediated by basic fibroblast growth factor and vascular endothelial cell growth factor. Differentiation, heterogeneity, and aneuploidy are regulated by protooncogenes and tumor suppressor genes.

29 A 22-year-old woman, who works as a secretary for an accounting firm, has noted a palpable nodule on the side of her neck for the past 3 months. On physical examination, there is a 2-cm, firm, nontender nodule involving the right lobe of the thyroid gland. A fine-needle aspiration biopsy specimen of the nodule shows cells consistent with carcinoma of the thyroid. No other family members are affected by this disorder. Which of the following would be considered most relevant in the woman's past medical history? □ (A) Chronic alcoholism □ (B) Ataxia telangiectasia □ (C) Radiation therapy in childhood □ (D) Blunt trauma from a fall □ (E) Exposure to arsenic compounds

29 (C) Radiation is oncogenic. Cancers of thyroid and bone often develop after radiation exposure; leukemias also can occur. Hepatocellular carcinomas can arise in cirrhosis caused by chronic alcoholism. Ataxia telangiectasia is an inherited syndrome that carries an increased risk of development of leukemias and lymphomas. Trauma is not a risk factor for development of cancer, although traumatic episodes often are recalled and irrationally associated with subsequent health problems. Arsenic exposure, which is uncommon, leads to lung and skin cancers.

29 A 36-year-old man is singing the blues on Beale Street in Memphis, Tennessee, from a month-long chronic cough with a low-grade fever. On physical examination, his temperature is 37.4°C. A chest radiograph shows a peripheral, 2.5-cm mass in the right lower lobe. A fine-needle aspirate of the mass shows inflammation with mononuclear cells, including macrophages that, with PAS stain, show the findings seen in the figure. Which of the following mechanisms best explains how this infection was contracted? □ (A) Mosquito bite on a trip to a wooded area □ (B) Transfusion of packed RBCs □ (C) Injection drug use with shared needles □ (D) Ingestion of contaminated dairy products □ (E) Birds roosting on his air conditioner

29 (E) The figure shows yeasts in macrophages in the coin lesion of the lung produced by a granulomatous inflammatory process. On chest radiographs, the lesion appears sharply demarcated and rounded like a Canadian or U.S. dollar coin. One cause for this process is infection with Histoplasma capsulatum. Bird droppings, especially from pigeons, are a rich source of dusts contaminated with H. capsulatum. Mosquitoes generally are not known as vectors for diseases that cause granulomatous inflammation. Fungal and mycobacterial infections are not acquired by transfusion or other parenteral routes, such as sharing intravenous needles. Contaminated milk is a source of Mycobacterium bovis, but this is a rare pulmonary infection.

3 A 20-year-old man who has multiple sexual partners and does not use barrier precautions comes to the physician complaining of a nontender ulcer on the penis that has been present for 1 week. On physical examination, the 0.6-cm lesion has a firm, erythematous base and sharply demarcated borders. The lesion is scraped, and darkfield examination is positive for spirochetes consistent with Treponema pallidum. Which of the following is most likely to be seen microscopically in the biopsy specimen? □ (A) Granulomatous inflammation with suppuration □ (B) Granulomatous inflammation with caseation □ (C) Acute inflammation with abscess formation □ (D) Perivascular inflammation with plasma cells □ (E) Gummatous inflammation

3 (D) Syphilitic chancres occur in the primary stage of syphilis and are characterized by lymphoplasmacytic infiltrates and by an obliterative endarteritis. Similar lesions also may appear with secondary syphilitic mucocutaneous lesions. Suppurative granulomas are typical of cat-scratch disease. Caseating granulomatous inflammation is more characteristic of tuberculosis or fungal infections. Acute inflammation with abscess formation is characteristic of bacterial infections such as gonorrhea. Gummatous inflammation can be seen in adults with tertiary syphilis or in congenital syphilis.

34 A 40-year-old man notices an increasing number of lumps in the groin and armpit. On physical examination, he has generalized nontender lymph node enlargement and hepatosplenomegaly. An inguinal lymph node biopsy specimen shows a malignant tumor of lymphoid cells. Immunoperoxidase staining of the tumor cells with antibody to BCL2 is positive in the lymphocytic cell nuclei. Which of the following mechanisms has most likely produced the lymphoma? □ (A) Increased tyrosine kinase activity □ (B) Lack of apoptosis □ (C) Gene amplifications □ (D) Reduced DNA repair □ (E) Loss of cell cycle inhibition

3 4 (B) Overexpression of the BCL2 gene prevents apoptosis, allowing accumulation of cells in lymphoid tissues. Increased tyrosine kinase activity results from mutations affecting the ABL oncogene. Gene amplifications typically affect the ERBB2 (HER2) and MYC oncogenes. Reduced DNA repair occurs in the inherited disorder xeroderma pigmentosa. Loss of cell cycle inhibition results from loss of tumor suppressor genes such as p53.

3 A 30-year-old woman who has had multiple sexual partners sees her physician because she has had vaginal bleeding and discharge for the past 5 days. On physical examination, she is afebrile. Pelvic examination shows an ulcerated lesion arising from the squamocolumnar junction of the uterine cervix. A cervical biopsy is performed. Microscopic examination reveals an invasive tumor containing areas of squamous epithelium, with pearls of keratin. In situ hybridization shows the presence of human papillomavirus type 16 (HPV-16) DNA within the tumor cells. Which of the following molecular abnormalities in this tumor is most likely related to infection with HPV-16? □ (A) Trapping of the RAS protein in a GTP-bound state □ (B) Increased expression of laminin receptor genes □ (C) Inability to repair DNA damage □ (D) Functional inactivation of the RB1 protein □ (E) Increased expression of epidermal growth factor receptor

3 (D) The oncogenic potential of human papillomavirus (HPV), a sexually transmissible agent, is related to products of two early viral genes—E6 and E7. E7 binds to RB protein to cause displacement of normally sequestered transcription factors, which nullifies tumor suppressor activity of the RB protein. E6 binds to and inactivates the p53 gene product. Trapping of GTP-bound RAS protein can occur in many tumors but is not related to HPV infection. Laminin receptor expression correlates with metastatic potential of a malignant neoplasm. Inability to repair DNA damage plays a role in some colon and skin cancers. Increased epidermal growth factor receptor expression is a feature seen in many pulmonary squamous cell carcinomas, and the related ERBB2 (HER2) receptor is seen in some breast carcinomas.

31 A 62-year-old man with a history of chronic alcoholism has noted a 6-kg weight loss over the past 5 months. Physical examination shows no masses or palpable lymphadenopathy. Laboratory studies include an elevated serum α-fetoprotein level. A stool guaiac test result is negative. Which of the following is the most likely diagnosis? □ (A) Prostatic adenocarcinoma □ (B) Pulmonary squamous cell carcinoma □ (C) Multiple myeloma □ (D) Pancreatic adenocarcinoma □ (E) Hepatocellular carcinoma

3 1 (E) α-Fetoprotein is a tumor marker for hepatocellular carcinomas and some testicular carcinomas. The serum prostate-specific antigen is a helpful marker for prostatic adenocarcinoma. Squamous cell carcinomas of any site do not have useful specific tumor markers. A serum immunoglobulin level with protein electrophoresis aids in the diagnosis of myeloma. Gastrointestinal tract adenocarcinomas, including those arising in the pancreas, may be accompanied by elevations in the serum carcinoembryonic antigen level.

32 A 49-year-old man experiences an episode of hemoptysis. On physical examination, he has puffiness of the face, pedal edema, and systolic hypertension. A chest radiograph shows a 5-cm mass of the right upper lobe of the lung. A fine-needle aspiration biopsy of this mass yields cells consistent with small-cell anaplastic carcinoma. A bone scan shows no metastases. Immunohistochemical staining of the tumor cells is most likely to be positive for which of the following? □ (A) Parathyroid hormone-related peptide □ (B) Erythropoietin □ (C) Corticotropin □ (D) Insulin □ (E) Gastrin

3 2 (C) This patient has Cushing syndrome resulting from ectopic corticotropin production by the tumor, a form of paraneoplastic syndrome common to small-cell carcinomas of the lung. Hypercalcemia from a parathormone-related peptide (PrP) is more typically associated with pulmonary squamous cell carcinomas. Erythropoietin production with polycythemia is more likely to be associated with a renal cell carcinoma. Insulin and gastrin production are most often seen in islet cell tumors of the pancreas.

32 A 25-year-old woman has had pelvic pain, fever, and vaginal discharge for 3 weeks. On physical examination, she has lower abdominal adnexal tenderness and a painful, swollen left knee. Laboratory studies show WBC count of 11,875/mm3 with 68% segmented neutrophils, 8% bands, 18% lymphocytes, and 6% monocytes. The patient receives antibiotic therapy and recovers. She undergoes a work-up for infertility 5 years later. Which of the following infectious agents is most likely to produce these findings? □ (A) Bacteroides fragilis □ (B) Campylobacter jejuni □ (C) Candida albicans □ (D) Entamoeba histolytica □ (E) Herpes simplex virus □ (F) Neisseria gonorrhoeae □ (G) Pseudomonas aeruginosa □ (H) Toxoplasma gondii

3 2 (F) This patient has pelvic inflammatory disease (PID), which may occur as a result of infection with Neisseria gonorrhoeae or Chlamydia trachomatis. Both organisms cause sexually transmitted diseases, and chronic inflammation may lead to PID. Complications of PID include peritonitis, adhesions with bowel obstruction, sepsis with endocarditis, meningitis, arthritis, and infertility. Of the remaining organisms listed, Candida can produce vaginitis with a curdlike discharge, but it does not typically produce PID. Herpes simplex virus can produce painful vesicles, usually on the external genitalia, and is often recurrent. The other listed organisms often are not present in the female genital tract.

33 During a routine health maintenance examination of a 40-year-old man, a stool guaiac test result was positive. A followup sigmoidoscopy showed a 1.5-cm, circumscribed, pedunculated mass on a short stalk, located in the upper rectum. Which of the following terms best describes this lesion? □ (A) Adenoma □ (B) Hamartoma □ (C) Sarcoma □ (D) Choristoma □ (E) Nevus

3 3 (A) A discrete small mass such as that described is probably benign. Adenomas arise from epithelial surfaces. A hamartoma is a rare benign mass composed of tissues usually found at the site of origin. A sarcoma is a malignant neoplasm arising in mesenchymal tissues. A choristoma is a benign mass composed of tissues not found at the site of origin. A nevus arises in the skin.

34 An 18-year-old woman gives birth at 33 weeks' gestation to a stillborn boy. Autopsy shows extensive periventricular cerebral necrosis with calcification and vascular thrombosis in the circle of Willis. Small areas of necrosis also appear in the heart and lung. Which of the following is most likely part of the life cycle of the infection in this stillborn fetus? □ (A) Cat □ (B) Flea □ (C) Pig □ (D) Sand fly □ (E) Tick □ (F) Triatomid bug

3 4 (A) Toxoplasmosis is the "T" in the TORCH mnemonic for congenital infections (Toxoplasmosis, Other infections, Rubella, Cytomegalovirus infection, and Herpes simplex). The Toxoplasma gondii organisms can cross the placenta to affect the fetus. The mother is typically asymptomatic. Cats are the natural hosts for T. gondii . Fleas can be vectors for infections, such as those caused by some Rickettsia species, and for the Black Death, caused by Yersinia pestis. The pig can be involved in the life cycle of Taenia solium and of Trichinella spiralis. The sand fly is a vector for Leishmania infections. Ticks can transmit typhus and Lyme disease. Triatomid bugs harbor Trypanosoma cruzi organisms, which cause Chagas disease.

35 A 32-year-old man has maculopapular and nodular skin lesions, mainly involving his face, elbows, wrists, and knees. The nodular lesions have slowly enlarged over the past 10 years and are now beginning to cause deformity. The lesions are not painful, but the patient has decreased to absent sensation in these areas. The figure shows an acid-fast stain of a biopsy specimen of a nodular lesion. Which of the following is the most likely diagnosis? □ (A) Anthrax □ (B) Chagas disease Robbins & Cotran Review of Pathology Pg. 135 □ (C) Leishmaniasis □ (D) Leprosy □ (E) Lyme disease □ (F) Onchocerciasis □ (G) Syphilis

3 5 (D) Leprosy, also known as Hansen's disease, is caused by the small, acid-fast organism Mycobacterium leprae, which chronically infects peripheral nerves and skin. This organism cannot be cultured in artificial media. Diagnosis is made by biopsy of a skin lesion. There are two polar forms of leprosy: tuberculoid and lepromatous. In the tuberculoid form, a delayed hypersensitivity reaction gives rise to granulomatous lesions that resemble tuberculosis. Acid-fast bacilli are rare in such lesions. In contrast, in the lepromatous form, shown in the figure, T cell immunity is markedly impaired, and granulomas are not formed. Instead, there are large aggregates of lipid-filled macrophages that are stuffed with acidfast bacilli. Leprosy is poorly transmissible through aerosols (not from direct contact); probably requires some genetic susceptibility; and, similar to most diseases throughout human history, is linked to poverty. Cutaneous anthrax, caused by Bacillus anthracis, produces a necrotic skin lesion with eschar at the site of inoculation. The reduviid bug carries Trypanosoma cruzi, which causes Chagas disease. Its bite may cause a localized area of skin erythema and swelling. Mucocutaneous ulcers may be seen with Leishmania braziliensis infection, which is transmitted via sand flies. The area of the tick bite that introduces Borrelia burgdorferi spirochetes, the cause of Lyme disease, may manifest erythema chronicum migrans. Onchocerciasis occurs as a result of infection with the filarial nematode Onchocerca volvulus and leads to formation of a subcutaneous nodule.

30 A study of transfusion-related infectious diseases determines that some blood donors seem to have acquired an infection via vertical transmission from mother to child. Laboratory testing strategies are devised to detect the most common of these infections and exclude such individuals as blood donors. As a consequence, which of the following infectious agents is most likely to be a significant cause for rejection as a blood donor later in life? □ (A) Escherichia coli □ (B) Hepatitis B virus □ (C) Plasmodium vivax □ (D) Candida albicans □ (E) Pneumocystis carinii

30 (B) Testing for hepatitis B and C is part of routine screening of blood donors. This form of transmission for hepatitis B is most common in developing nations. Escherichia coli can be a congenital infection, but it leaves no major significant lasting sequelae in infants who survive. Malaria, candidal infection, and pneumocystosis are not congenital infections.

30 An epidemiologic study is performed to assess risks for cervical carcinoma. The cells from cervical lesions in a population of women are analyzed. Binding of certain viral proteins to pRB is found in patients in whom dysplastic cells are present. Viral proteins from which of the following are most likely to bind pRB, increasing the risk for dysplasia? □ (A) Cytomegalovirus □ (B) Epstein-Barr virus □ (C) Herpes simplex virus □ (D) Hepatitis B virus □ (E) HIV □ (F) Human papillomavirus □ (G) JC papovavirus

30 (F) Human papillomavirus (HPV) types 16, 18, and 31 encode proteins that bind p53 with high affinity, resulting in loss of tumor suppressor activity. Seventy-five percent to nearly 100% of squamous epithelial dysplasias and carcinomas of the cervix are associated with HPV infection. Cytomegalovirus and herpes simplex virus do not participate directly in carcinogenesis. Epstein-Barr virus is associated with some malignant lymphomas and nasopharyngeal carcinomas. Hepatitis B virus is associated with hepatocellular carcinomas arising in the setting of regeneration in chronic liver injury. HIV does not affect pRB, but the loss of immune regulation promotes development of lymphomas and Kaposi sarcoma. The JC papovavirus is associated with development of progressive multifocal leukoencephalopathy.

31 In October 1347, a Genoese trading ship returning from the Black Sea docked at Messina, Sicily. The ship's crew had been decimated by an illness marked by a short time course of days from onset of inguinal lymph node enlargement with overlying skin ulceration to prostration and death. A small, ulcerated pustule ringed by a rosy rash was seen on the lower extremities of some of the crew. Within days, more than half of the population of the port city had died. Which of the following vectors was most likely responsible for the rapid spread of this disease? □ (A) Mosquitoes □ (B) Rats □ (C) Sand flies □ (D) Cats □ (E) Ticks

31 (B) This incident marks the first appearance of the Black Death in Europe, a disease that persisted during the 14th and 15th centuries. The plague spread through Italy and across the European continent. By the following spring, it had reached as far north as England, and within 5 years, it had killed 25 million people, one third of the European population. Rodents form the reservoir of infection. Flea bites and aerosols transmit the infection very efficiently. The causative organism, Yersinia pestis, secretes a plasminogen activator that promotes its spread. Plague was endemic in East Asia at the beginning of the 20th century and was carried to San Francisco. Seeking to avoid a panic that could be bad for business and tourism, California's governor at the time did not enforce a quarantine. As a consequence, plague is endemic in wild rodents in the western United States, but it accounts for only occasional sporadic human infections. Mosquitoes are best known as vectors of malaria; sand flies, of leishmaniasis; cats, of toxoplasmosis; and ticks, of Lyme disease.

33 A 9-year-old girl has developed a mild febrile illness with a sore throat over the past 2 days. On physical examination, her temperature is 38.4°C, and she has a mild pharyngitis. The girl's symptoms subside in 1 week without therapy. Over the next 2 months, she has increasing right-sided facial drooping with inability to close the right eye. Which of the following infectious organisms is most likely to produce these findings? □ (A) Cryptococcus neoformans □ (B) Cytomegalovirus □ (C) Listeria monocytogenes □ (D) Poliovirus □ (E) Toxoplasma gondii

33 (D) Poliomyelitis is an enterovirus spread through fecal-oral contamination. The virus infects the oropharynx first. It then spreads to spinal cord anterior horn cells and bulbar nuclei to produce the paralysis typical of polio. In places where vaccination is routinely available, this disease is rare. Cryptococcosis most often involves the lungs and meninges. Cytomegalovirus infection can be congenital; in immunocompromised adults, it can involve many organs, principally the gastrointestinal tract, brain, and lungs. Listeriosis is most often acquired via contaminated food or water; in most adults, it produces mild diarrheal illness, but in some adults and children, and in fetuses, it can produce meningitis or dissemination with microabscess (microgranuloma) formation. Toxoplasmosis can be a congenital infection. In immunocompromised adults, it can produce inflammation in multiple tissues, but most often, it causes chronic abscessing inflammation in brain.

38 A 6-month-old infant has abrupt onset of vomiting followed by profuse, watery diarrhea. On physical examination, the infant has a temperature of 38.3°C. Development is normal for age, and the only abnormal finding is poor skin turgor. Laboratory studies show serum Na+ of 153 mmol/L, K+ of 4.4 mmol/L, Cl− of 111 mmol/L, CO2 of 27 mmol/L, and glucose of 70 mg/dL. Examination of a stool specimen shows mucus, but no RBCs or WBCs. Which of the following mechanisms accounts for this diarrhea? □ (A) Decreased absorption of sodium and water □ (B) Increased secretion of potassium and water by epithelial cells Robbins & Cotran Review of Pathology Pg. 136 □ (C) Presence of Yop virulence plasmid □ (D) Lysis of colonic epithelial cells □ (E) Decreased breakdown of lactose to glucose and galactose

38 (A) Rotavirus, an encapsulated RNA virus, is a major cause of diarrhea in infancy. The villous destruction with atrophy leads to decreased absorption of sodium and water. The development of antibodies from secretory immunity in the bowel to rotavirus surface antigens causes older children and adults to be relatively resistant to rotavirus infection. Such antibodies are present in maternal milk and confer some degree of resistance to infants who breastfeed. Rotavirus infection occurs worldwide. By the age of 3 years, virtually every individual has been infected by rotaviruses at least once. Most rotavirus infections are subclinical or cause mild gastrointestinal illnesses that do not require hospitalization. The first infection is the most likely to be symptomatic; subsequent infections are often mild or asymptomatic. Many enteroviruses also produce diarrhea by inhibiting the intestinal absorption of intraluminal sodium and water. Most older children and adults have immunity. Cholera is the result of secretion of an exotoxin by the Vibrio cholerae organism, which potentiates the epithelial cell production of adenylate cyclase and causes secretory diarrhea with sodium chloride and water loss. The Yop plasmid confers infectivity to Yersinia organisms. Amebae can lyse epithelium, and the diarrhea can be bloody. Decreased breakdown of lactose occurs in disaccharidase deficiency and gives rise to an osmotic diarrhea.

35 A 70-year-old woman reported a 4-month history of a 4-kg weight loss and increasing generalized icterus. On physical examination, she is afebrile, and her blood pressure is 130/80 mm Hg. An abdominal CT scan shows a 5-cm mass in the head of the pancreas. Fine-needle aspiration of the mass is performed. On molecular analysis, the neoplastic cells from the mass show continued activation of cytoplasmic kinases. Which of the following oncogenes is most likely to be involved in this process? □ (A) MYC □ (B) APC □ (C) RAS □ (D) ERBB2 □ (E) sis

35 (C) The RAS oncogene is the most common oncogene involved in the development of human cancers. Mutations of the RAS oncogene reduce GTPase activity, and RAS is trapped in an activated GTP-bound state. RAS then signals the nucleus through cytoplasmic kinases. The MYC oncogene is a transcriptional activator that is overexpressed in many tumors. The APC gene can cause activation of the WNT signaling pathway. The ERBB2 oncogene encodes growth factor receptors that are amplified in certain tumors. The sis oncogene encodes platelet-derived growth factor receptor-β, which is overexpressed in certain astrocytomas.

36 A 9-year-old child who is living in a mud hut in northeastern Brazil has a sore on her face. She is taken to the physician, and physical examination shows an indurated area of erythema and swelling just lateral to the left eye, accompanied by posterior cervical lymphadenopathy. She has unilateral painless edema of the palpebrae and periocular tissues. Two days later, she has malaise, fever, anorexia, and edema of the face and lower extremities. On physical examination 1 week later, there is hepatosplenomegaly and generalized lymphadenopathy. Which of the following pathologic findings is most likely to develop in this patient? □ (A) Cerebral abscesses □ (B) Chronic arthritis □ (C) Dilated cardiomyopathy □ (D) Meningitis □ (E) Mucocutaneous ulcers □ (F) Myositis □ (G) Paranasal bony destruction

36 (C) This child is infected with Trypanosoma cruzi, resulting in Chagas disease. The vector is the reduviid (triatomid) bug. The organisms can damage the heart by direct infection or by inducing an autoimmune response that affects the heart because of the existence of cross-reactive antigen. Acute myocarditis rarely occurs, but most deaths in acute Chagas disease are due to heart failure. In 20% of infected individuals, cardiac failure can occur 5 to 15 years after the initial infection. The affected heart is enlarged, and all four chambers are dilated. A cerebral abscess or acute meningitis is typically a complication of a bacterial infection with septicemia. Chronic arthritis can be seen in Lyme disease, which is transmitted by deer ticks. Mucocutaneous ulcers may be seen in Leishmania braziliensis infection, which is transmitted via sand flies. Myositis can be the result of infection with Trichinella spiralis, which is acquired from poorly cooked pork. Paranasal sinus infection may be caused by Mucor circinelloides.

36 A 23-year-old woman has noted a nodule on the skin of her upper chest. She reports that the nodule has been present for many years and has not changed in size. On physical examination, there is a 0.5-cm, dark red, nontender, raised nodule with a smooth surface. Which of the following is the most likely diagnosis? □ (A) Adenoma □ (B) Fibroadenoma □ (C) Hamartoma □ (D) Hemangioma □ (E) Leiomyoma □ (F) Lipoma □ (G) Melanoma □ (H) Nevus

36 (D) The small, discrete nature of this mass and its unchanged size suggest a benign neoplasm. The red color suggests vascularity. A hemangioma is a common benign lesion of the skin. Adenomas arise in glandular epithelium, such as the colon. Fibroadenomas arise in the breast. A hamartoma contains a mixture of cell types common to a tissue site. Leiomyomas, which are white, arise from smooth muscle and are most common in the uterus. Lipomas are yellow fatty tumors. Melanomas are malignant and tend to increase in size quickly; many are darkly pigmented. The benign counterpart to the melanoma is the nevus, which is quite common; nevi are usually light brown.

37 A 50-year-old resident of Phoenix, Arizona, has a cough that has persisted for 1 month. On physical examination, his temperature is 38.1°C. A chest radiograph shows a 3.5-cm opacity with central cavitation in the right apical region. An open lung biopsy is performed to exclude cancer. Microscopic examination of the biopsy specimen shows caseating granulomatous inflammation containing 60-μm spherules filled with smaller, rounded structures. Which of the following infectious organisms is most likely to produce these findings? □ (A) Aspergillus fumigatus □ (B) Coccidioides immitis □ (C) Histoplasma capsulatum □ (D) Mycobacterium tuberculosis □ (E) Pseudomonas aeruginosa □ (F) Pneumocystis carinii □ (G) Staphylococcus aureus □ (H) Toxoplasma gondii

37 (B) Inhaling the arthrospores of Coccidioides immitis can lead to coccidioidomycosis. This disease is endemic to the southwestern United States. The infection typically results in granuloma formation, but most individuals have subclinical infections. About 10% may be symptomatic with respiratory symptoms, including cough and pleuritic chest pain. Dissemination to extrapulmonary sites occurs in only 1% of cases. Aspergillus organisms have branching septate hyphae. Histoplasma capsulatum organisms are about 2 to 4 μ (roughly the size of the endospores of C. immitis) and are often found within macrophages. Mycobacterium tuberculosis organisms are identified with acid-fast stains to highlight the rodlike bacterial shape. Pseudomonas aeruginosa is a gram-negative rodlike bacterium. Pneumocystis carinii produces 7- μ cysts that are seen with Gomori methenamine silver stain. Staphylococcus aureus grows as clusters of gram-positive cocci. Toxoplasma gondii produces pseudocysts filled with tachyzoites, but the pseudocyst does not have a thick wall. T. gondii is a rare pulmonary infection that produces small, focal, mixed inflammatory exudates.

37 A 63-year-old man sees the physician because of cough and hemoptysis. He has a 65-pack-year history of smoking. A chest CT scan shows a 5-cm right hilar mass. Bronchoscopy is performed, and lung biopsy specimens show small-cell anaplastic lung carcinoma. His family history shows three first-degree maternal relatives who developed leukemia, sarcoma, and carcinoma before age 40 years. Which of the following genes is most likely to have undergone mutation to produce these findings? □ (A) APC (tumor suppressor gene) □ (B) BCL2 (anti-apoptosis gene) □ (C) ERBB2 (growth factor receptor gene) □ (D) K-RAS (GTP-binding protein gene) □ (E) NF1 (GTPase-activating protein) □ (F) p53 (DNA damage response gene)

37 (F) p53 is the most common target for genetic alterations in human neoplasms. Most are sporadic mutations, although some are inherited. The inheritance of one faulty p53 suppressor gene predisposes to a "second hit" that eliminates the remaining p53 gene. Homozygous loss of the p53 genes dysregulates the repair of damaged DNA, predisposing individuals to multiple tumors, as in this case. The APC gene is mutated in sporadic colon cancers and in familial polyposis coli. The BCL2 gene is mutated in some non-Hodgkin lymphomas. The ERBB2 gene is one of the EGF receptor family members amplified in some breast cancers. The EGF mutation is most often seen in squamous cell carcinomas of the lung. K-RAS mutations are present in many cancers, but not typically lymphoid malignancies. The NF1 gene mutation is seen in neurofibromatosis type 1.

38 A 30-year-old man has a pheochromocytoma of the left adrenal gland; a sibling had a cerebellar hemangioblastoma. He undergoes adrenalectomy, and on microscopic examination there is extensive vascularity of the neoplasm. Mutational analysis of the neoplastic cells shows that both allelic copies of a gene have been lost, so that a protein that binds to hypoxia-induced factor 1 alpha is no longer ubiquinated, but instead translocates to the nucleus and activates transcription of VEGF. Which of the following genes is most likely mutated in this man? □ (A) APC □ (B) BCL2 □ (C) EGF □ (D) HER2 □ (E) HST1 □ (F) MYC □ (G) VHL

38 (G) Angiogenesis is a key feature of neoplasms because the growing tumor needs a blood supply, and up-regulation of factors such as VEGF and FGF help to keep the cancer growing. The von Hippel-Lindau (VHL) gene acts as a tumor suppressor, and it normally produces a protein that binds to hypoxia-induced factor 1 alpha so that it is cleared. VHL mutation leads to loss of this binding protein and activation of angiogenesis factors. Individuals with VHL syndrome have various neoplasms, including pheochromocytomas, renal cell carcinomas, and hemangioblastomas. The other listed genes have products that do not directly act on angiogenesis pathways.

39 In a clinical trial, patients diagnosed with malignant melanoma are treated by infusion of autologous CD8+ T cells grown in vitro. These CD8+ T cells are known to kill melanoma cells, but not normal cells. Which of the following target antigens is most likely recognized by these CD8+ T cells? □ (A) Class I MHC molecules with a melanoma cell peptide □ (B) Class I MHC molecules with a peptide from normal melanocytes and melanoma cells □ (C) Class I MHC molecules plus a peptide derived from carcinoembryonic antigen □ (D) Class II MHC molecules with a melanoma cell peptide □ (E) Class II MHC molecules with a peptide from normal melanocytes and melanoma cells □ (F) Class II MHC molecules with laminin receptors on melanoma cells

39 (A) All human nucleated cells express MHC class I antigens. CD8+ T cells recognize peptides presented by MHC class I antigens. In many tumors, especially melanomas, the tumor cells produce peptides that can be presented by MHC class I molecules. Such tumor-specific peptides are not produced by other cells, so the CD8+ T cells specific for such peptides lyse melanoma cells, but not normal melanocytes or other normal cells.

39 A 5-year-old girl has a blotchy, reddish brown rash on her face, trunk, and proximal extremities that developed over the course of 3 days. On physical examination, she has 0.2- to 0.5-cm ulcerated lesions on the oral cavity mucosa and generalized lymphadenopathy. A cough with minimal sputum production becomes progressively worse over the next 3 days. Which of the following viruses is most likely to produce these findings? □ (A) Mumps □ (B) Varicella zoster □ (C) Rubella □ (D) Epstein-Barr □ (E) Rubeola

39 (E) The rash and the Koplik spots on the buccal mucosa are characteristic findings in measles (rubeola), a childhood infection. It occurs only sporadically when immunizations have been administered to a large part of the population. The severity of the illness varies, and a measles pneumonia may complicate the course of the disease, which in some cases can be life-threatening. Mumps produces parotitis and orchitis. Varicella-zoster virus infections in children manifest as chickenpox. Rubella, also called German measles, is a much milder infection than rubeola. Mononucleosis, which results from Epstein-Barr virus infection, is more likely to occur in adolescence.

4 A 24-year-old man has a fever and a runny nose, sneezing, and coughing that have worsened over the past 4 days. The symptoms abate, and he has sequelae. This infection is most likely to be promoted by binding of which of the following organisms to intercellular adhesion molecule-1 (ICAM-1)? □ (A) Mycoplasma pneumoniae □ (B) Haemophilus influenzae □ (C) Rhinovirus □ (D) Epstein-Barr virus □ (E) Neisseria meningitidis

4 (C) Rhinovirus binds to intercellular adhesion molecule-1 (ICAM-1) and accounts for 60% of common colds. Mycoplasma pneumoniae also accounts for some colds, but this agent does not bind to ICAM-1 receptors on host cells. Haemophilus influenzae can produce sinusitis, otitis media, and bronchopneumonia. Infection with Epstein-Barr virus can produce pharyngitis with infectious mononucleosis, but the course tends to extend over weeks. Neisseria meningitidis infections, which produce meningitis, may begin as a mild pharyngitis, but these infections can have a very rapid course over 1 or 2 days.

4 An experiment involving carcinoma cells grown in culture studies the antitumor surveillance effects of the innate immune system. These carcinoma cells fail to express MHC class I antigens. It is observed, however, that carcinoma cells are lysed in the presence of one type of immune cell that has been activated by IL-2. Which of the following immune cells is most likely to function in this manner? □ (A) CD4+ lymphocyte □ (B) CD8+ lymphocyte □ (C) Macrophage □ (D) Neutrophil □ (E) NK cell □ (F) Plasma cell

4 (E) Several types of immune cells can recognize and help destroy tumor cells. Tumor antigens that are displayed via MHC class I molecules can be recognized by cytotoxic CD8+ cells. Many cancers do not display their antigens well, however, and when MHC class I molecules are not displayed, NK cells are triggered to target these cells for lysis. Macrophages may work in concert with CD8+ cells and NK cells to phagocytize and kill tumor cells when up-regulated by interferon-γ. CD4+ "helper" cells do not play a direct anticancer role. Neutrophils are ineffective against cancer cells, but may be attracted to areas of tumor necrosis. Plasma cells may produce antibodies directed against tumor antigens, but such antibodies are ineffective in controlling tumors.

45 In an experiment, cells from human malignant neoplasms explanted into tissue culture medium continue to replicate. This allows development of "immortal" tumor cell lines that are extremely useful for study of tumor biology and responses to therapeutic modalities. Which of the following molecular alterations that endows these tumor cells with limitless replicative ability in vivo and in vitro is most likely to be observed? □ (A) Activation of telomerase □ (B) Activation of cyclin genes □ (C) Inhibition of cyclin activators □ (D) Activation of vascular endothelial growth factor □ (E) Activation of BCL2 gene □ (F) Inability to repair errors in DNA replication

4 5 (A) Chromosomal telomere shortening in normal human cells limits their replicative potential and gives rise to replicative senescence. This occurs because most somatic cells lack the enzyme telomerase. By contrast, 90% or more human tumor cells show activation of telomerase, explaining continued tumor growth in the body and "immortalized" cell lines. All other pathways listed cannot affect telomerase shortening, which is the rate-limiting step in indefinite replication of cells.

40 A 62-year-old man has had several episodes of hematuria in the past week. On physical examination, there are no abnormal findings. A urinalysis shows 4+ hematuria, and cytologic examination of the urine shows that atypical cells are present. The urologist performs a cystoscopy and observes a 4-cm sessile mass with a nodular, ulcerated surface in the dome of the bladder. Which of the following terms best describes this lesion? □ (A) Papilloma □ (B) Carcinoma □ (C) Adenoma □ (D) Sarcoma □ (E) Fibroma

40 (B) A large, irregular, ulcerated mass such as that described is most likely malignant, and the epithelium of the bladder gives rise to carcinomas. A papilloma is a benign, localized mass that has an exophytic growth pattern. An adenoma is a benign epithelial neoplasm of glandular tissues. A sarcoma is derived from cells of mesenchymal origin; sarcomas are much less common than carcinomas. A fibroma is a benign mesenchymal neoplasm.

41 A 66-year-old man with chronic cough has an episode of hemoptysis. On physical examination, there are no abnormal findings. A chest radiograph shows a 6-cm mass in the right lung. A sputum cytologic analysis shows cells consistent with squamous cell carcinoma. Metastases from this neoplasm are most likely to be found at which of the following sites? □ (A) Chest wall muscle □ (B) Splenic red pulp □ (C) Hilar lymph nodes □ (D) Vertebral bone marrow □ (E) Cerebrum

41 (C) Carcinomas metastasize through lymphatics most often, usually to regional nodes first. Hematogenous metastases are possible, however. About half of all cerebral metastases arise from the lung. Soft-tissue metastases are rare, as are splenic metastases.

42 A 69-year-old woman has experienced increasing malaise and a 10-kg weight loss over the past year. She dies of massive pulmonary thromboembolism. The gross appearance of the liver at autopsy is shown. Which of the following statements best characterizes the process that led to the patient's death? □ (A) A liver biopsy would have shown a dysplasia □ (B) This is a multifocal hepatic adenoma Robbins & Cotran Review of Pathology Pg. 110 □ (C) A hepatocellular carcinoma has invaded locally □ (D) Colonic adenocarcinoma with metastases was present □ (E) The lesions should have been resected

42 (D) The figure shows the appearance of metastatic lesions from a malignant neoplasm with multiple tumor masses present. Dysplastic lesions do not produce large masses. Although some benign tumors, such as leiomyomas of the uterus, can be multiple, this is not the rule in the liver, and hepatic adenomas are rare. Although hepatocellular carcinomas can have "satellite" nodules, widespread nodules such as those seen in the figure are more characteristic of metastases. Resection of multiple metastases is usually fruitless.

43 A 75-year-old woman has reported a change in the caliber of her stools during the past month. On physical examination, there are no abnormal findings, but a stool sample is positive for occult blood. A colonoscopy shows a constricting mass involving the lower sigmoid colon, and the patient undergoes a partial colectomy. Which of the following techniques used during surgery can best aid the surgeon in determining whether the resection is adequate to reduce the probability of a recurrence? □ (A) Fine-needle aspiration □ (B) Serum carcinoembryonic antigen assay □ (C) Frozen section □ (D) Electron microscopy □ (E) Flow cytometry

43 (C) The rapid frozen section of resection margins helps to determine whether enough of the colon has been resected. Fine-needle aspiration is used for preoperative diagnosis. Serum tumor markers may aid in preoperative diagnosis or postoperative follow-up of neoplasms. Electron microscopy requires at least 1 day to perform and helps to determine the cell type. Flow cytometry can be performed in several hours, but it is useful mainly for prognostic information and is not a "stat" procedure.

44 A 60-year-old woman has noted a feeling of pelvic heaviness for the past 6 months. On physical examination, there is a nontender lower abdominal mass. An abdominal ultrasound scan shows a 12-cm solid mass in the uterine wall. A total abdominal hysterectomy is performed, and on removal, the mass has the microscopic appearance of a well-differentiated leiomyosarcoma. One year later, a chest radiograph shows a 4-cm nodule on the right lower lung. A biopsy specimen of the nodule shows a poorly differentiated sarcoma. The patient's past medical history indicates that she has smoked cigarettes most of her adult life. Which of the following best explains these findings? □ (A) Development of a second primary neoplasm □ (B) Inheritance of a defective RB gene □ (C) Continued cigarette smoking by the patient □ (D) Loss of an oncogene □ (E) Metastasis from an aggressive subclone of the primary tumor

44 (E) Although neoplasms begin as monoclonal proliferations, additional mutations occur over time, leading to subclones of neoplastic cells with various properties. This subcloning may allow metastases, greater invasiveness, resistance to chemotherapy, and morphologic differences to occur. Because sarcomas of the lung are rare, the lung mass is statistically a metastasis. Inheritance of a mutant RB gene is most likely to lead to childhood retinoblastomas. Pulmonary sarcomas are not related to smoking tobacco. Loss of tumor suppressor genes, not oncogenes, is related to tumor development.

46 An investigational study reviews cells from patients who had hereditary nonpolyposis colon cancer. The patients typically developed multiple lesions of the colon during middle age. Molecular analysis of the cells from the lesions shows changes in hPMS1, hPMS2, and hMLH1 genes. Which of the following principles of carcinogenesis is best illustrated by this study? □ (A) Tumor initiators are mutagenic □ (B) Tumor promoters induce proliferation □ (C) Many oncogenes are activated by translocations □ (D) Inability to repair DNA is carcinogenic □ (E) Carcinogenesis is a multistep process

46 (D) Patients with hereditary nonpolyposis colon carcinoma (HNPCC) inherit one defective copy of mismatch repair genes. Several human mismatch repair genes are involved in the development of HNPCC. Mismatch repair defects have microsatellite instability. Microsatellites are tandem repeats found throughout the genome. Normally, the length of these microsatellites remains constant. In HNPCC, these satellites are unstable and increase or decrease in length. Although HNPCC accounts for only 2% to 4% of all colon carcinomas, microsatellite instability can be detected in about 15% of all sporadic colon carcinomas. Mutations in mismatch repair genes can be detected by the presence of microsatellite instability. The other listed options are not characteristic of HNPCC.

47 A 38-year-old woman has abdominal distention that has been worsening for the past 6 weeks. An abdominal CT scan shows bowel obstruction caused by a 6-cm mass in the jejunum. At laparotomy, a portion of the small bowel is resected. Microscopic examination shows that the mass is a Burkitt lymphoma. Flow cytometry analysis of a portion of the tumor shows a high S phase. Mutational activation of which of the following nuclear oncogenes is most likely to be present in this tumor? □ (A) ERBB2 □ (B) p53 □ (C) RAS □ (D) MYC □ (E) APC

47 (D) The MYC oncogene is commonly activated in Burkitt lymphoma because of a t(8;14) translocation. The MYC gene binds DNA to cause transcriptional activation of growth-related genes such as that for cyclin D1, resulting in activation of the cell cycle. ERBB2 (also known as HER2) encodes growth factor receptor located on the cell surface. p53 and APC are tumor suppressor genes that are inactivated in many cancers, including colon cancer (APC). RAS oncogene encodes a GTP-binding protein that is located under the cell membrane.

48 A 55-year-old woman has felt an enlarging lump in her left breast for the past year. A hard, irregular 5-cm mass fixed to the underlying chest wall is palpable in her left breast. Left axillary nontender lymphadenopathy is noted. There is no hepatosplenomegaly. A chest CT scan reveals multiple bilateral pulmonary "cannonball" nodules. A left breast biopsy is performed, and on microscopic examination shows high-grade infiltrating ductal carcinoma. The appearance of the nodules in her lungs is most likely related to which of the following? □ (A) Proximity of the breast carcinoma to the lungs □ (B) Lymphatic connections between the breast and the pleura □ (C) Internal mammary artery invasion by carcinoma cells □ (D) Pulmonary chemokines that bind carcinoma cell chemokine receptors □ (E) Overexpression of laminin receptors on carcinoma cell surfaces □ (F) Overexpression of estrogen receptors within the carcinoma cell nuclei

48 (D) There is increasing evidence that localization of cancer metastases is influenced by the expression of chemokine receptors by cancer cells and elaboration of their ligands (chemokines) by certain tissues. In the case of breast cancer, the carcinoma cells express CXCR4 chemokines. Vascular, lymphatic, or basement membrane invasion is required for metastases, but these characteristics do not dictate accurately the location of metastases.

49 A 33-year-old woman undergoes a routine physical examination as part of health maintenance screening. There are no abnormal findings. A Pap smear is obtained as part of the pelvic examination. Cytologically, the cells obtained on the smear from the cervix show severe epithelial dysplasia. Which of the following statements best explains the significance of these findings? □ (A) The lesion could progress to invasive cervical carcinoma □ (B) An ovarian teratoma is present □ (C) There has been regression of a cervical carcinoma □ (D) Antibiotic therapy would cure the lesion □ (E) Female relatives are at risk of acquiring the same condition

49 (A) Epithelial dysplasias, especially severe dysplasias, can be precursors of carcinomas. This is a key reason for Pap smear screening; the incidence of cervical carcinoma decreases when routine Pap smears are performed. Teratomas show well-differentiated elements derived from all the germ cell layers, and they do not manifest as epithelial dysplasias. Severe dysplasias are not amenable to antibiotic therapy. Cervical dysplasias are not hereditary. Regression of a malignancy is rare.

5 A 44-year-old woman sees her physician because she feels lumps in the right axilla. The physician notes right axillary lymphadenopathy on physical examination. The nodes are painless but firm. Which of the following is the most likely diagnosis? □ (A) Ductal carcinoma of the breast □ (B) Acute mastitis with breast abscess □ (C) Leiomyosarcoma of the uterus □ (D) Cerebral glioblastoma multiforme □ (E) Squamous dysplasia of the larynx

5 (A) Lymphatic spread, especially to regional lymph nodes draining from the primary site, is typical of a carcinoma. Infection from a breast abscess can spread to the lymph nodes, but the resulting nodal enlargement is typically associated with pain—a cardinal sign of acute inflammation. Sarcomas uncommonly metastasize to lymph nodes. Central nervous system (CNS) malignancies rarely metastasize outside of the CNS. Dysplasias do not metastasize because they are not malignancies.

5 About 10 months after returning to the United States from a vacation to the Costa del Sol near Barcelona, a 45-year-old man experiences malaise and fatigue, which slowly become more noticeable over a 2-month period. He develops occasional diarrhea and a low-grade fever, with abdominal discomfort that worsens over the next month. On physical examination, his vital signs include temperature of 38.3°C, pulse of 81/min, respirations of 17/min, and blood pressure of 130/85 mm Hg. He has pronounced splenomegaly, an increased liver span, and generalized lymphadenopathy. Laboratory studies show hemoglobin, 11.8 g/dL; hematocrit, 34.9%; platelet count, 89,000/mm3; and WBC count, 3350/mm3 with 29% segmented neutrophils, 5% bands, 48% lymphocytes, and 18% monocytes. His total serum protein is 7.6 g/dL, albumin is 3.2 g/dL, AST is 67 U/L, ALT is 51 U/L, alkaline phosphatase is 190 U/L, and total bilirubin is 1.3 mg/dL. A stool sample is negative for occult blood. Which of the following is the most likely diagnosis? □ (A) Borreliosis □ (B) Echinococcosis □ (C) Leishmaniasis □ (D) Lyme disease □ (E) Schistosomiasis □ (F) Tuberculosis □ (G) Typhus

5 (C) Visceral leishmaniasis (kala-azar) is caused by protozoa in the Leishmania donovani complex. Of these, only L. donovani infantum is endemic to southern Europe and the Mediterranean area. It is transmitted to humans by the sandfly (Phlebotomus). Pancytopenia implies bone marrow involvement, possibly enhanced by the enlarged spleen, and the liver function abnormalities suggest liver involvement. Borreliosis causes relapsing fever and is transmitted via body lice. Echinococcal disease is caused by ingestion of tapeworm eggs and can lead to cyst formation in visceral organs. Borrelia burgdorferi infection is transmitted via ticks and can cause Lyme disease, characterized by erythema chronicum migrans, meningoencephalitis, and chronic arthritis. Schistosomiasis, which is transmitted via snails, can produce hepatic cirrhosis (Schistosoma mansoni or Schistosoma japonicum) or bladder disease (Schistosoma hematobium). It would be unusual for a case of tuberculosis not to start with pulmonary signs or symptoms before becoming disseminated. Typhus is a louse-borne rickettsial disease with skin rash that may proceed to skin necrosis.

52 A 35-year-old man with a family history of colon carcinoma undergoes a surveillance colonoscopy. It reveals hundreds of polyps in the colon, and two focal 0.5-cm ulcerated areas. A biopsy specimen from an ulcer reveals irregularly shaped glands that have penetrated into the muscular layer. Which of the following molecular events is believed to occur very early in the evolution of his colonic disease process? □ (A) Mutations in mismatch repair genes. □ (B) Inability to hydrolyze GTP-bound RAS □ (C) Activation of the WNT signaling pathway □ (D) Loss of heterozygosity affecting the p53 gene □ (E) Translocation of BCL2 from mitochondria to cytoplasm

5 2 (C) The patient has a classic history of familial adenomatous polyposis with numerous adenomatous polyps and malignant transformation. The earliest event in the APC → adenocarcinoma sequence is loss of APC gene function. This prevents the destruction of β-catenin in the cytoplasm, which translocates to the nucleus and coactivates transcription of several genes. The APC → β-catenin are components of the WNT signaling pathway. Mutations in mismatch repair genes give rise to hereditary nonpolyposis colon cancer syndrome from loss of ability to repair DNA damage. Loss of cell cycle G1 arrest occurs with p53 loss late in the sequence. The BCL2 gene is not involved in the transition from adenoma to carcinoma. RAS activation occurs after the sequence is initiated by the APC (gatekeeper) gene.

53 A 56-year-old woman has had vaginal bleeding for 1 week. Her last menstrual period was 10 years ago. On physical examination, a lower abdominal mass is palpated. An abdominal CT scan shows a 6-cm mass in the left ovary. A total abdominal hysterectomy is performed. Microscopically, the ovarian mass is a granulosa-theca cell tumor. The patient also is found to have an endometrial carcinoma, which resulted from increased estrogen production by the ovarian mass. Which of the following best describes the relationship between these two neoplasms? □ (A) Promotion of carcinogenesis □ (B) Tumor heterogeneity □ (C) Paraneoplastic syndrome □ (D) Genetic susceptibility to tumorigenesis □ (E) Mutation of a tumor suppressor gene

5 3 (A) Estrogen, similar to many other hormones and drugs, by itself is not carcinogenic, but it is responsible for stimulation of endometrial growth (hyperplasia), which has a promoting effect when mutations occur to produce carcinoma. Tumor heterogeneity does not refer to two separate kinds of neoplasms; it refers to heterogeneity with a given tumor or metastasis. A paraneoplastic syndrome results from ectopic secretion of a hormone by tumor (e.g., lung cancer cells producing corticotropin). Inherited susceptibility can never be completely excluded when an individual has two tumors; this can occur in patients with inherited mutations in the p53 gene. In this case, however, there is a clear hormonal basis for the second tumor. Faulty tumor suppressor genes are not involved in hormonal promotion of a neoplasm.

57 An epidemiologic study of cancer deaths recorded in the last half of the 20th century is conducted. The number of deaths for one particular cancer increased markedly in developed nations. In 1998, more than 30% of cancer deaths in men and more than 24% of cancer deaths in women were caused by this neoplasm. Which of the following neoplasms was most likely identified by this study? □ (A) Breast carcinoma □ (B) Bronchogenic carcinoma □ (C) Cervical squamous cell carcinoma □ (D) Adenocarcinoma of the colon □ (E) Melanoma of the skin □ (F) Prostatic adenocarcinoma

5 7 (B) Incidence of lung cancers increased dramatically in the 20th century because of the popularity of cigarette smoking. As the number of individuals in a population who smoke increases, so do the number of cases of lung cancers. Some cancers of the urinary tract, oral cavity, esophagus, and pancreas also are causally related to smoking. Breast, prostate, and colon cancers remain common in developed nations, but the number of cases has not increased sharply. Pap smear screening markedly decreases numbers of cases of cervical cancers. There has been a marked increase in melanomas, but there are still far fewer cases of melanomas than of lung cancers.

58 The mother of a 5-year-old boy notices that his abdomen is enlarged. On physical examination, the physician palpates an ill-defined abdominal mass. An abdominal CT scan shows a 9-cm mass in the region of the right adrenal gland. The mass is removed and microscopically appears to be a neuroblastoma. Cytogenetic analysis of tumor cells shows many Robbins & Cotran Review of Pathology Pg. 114 double minutes and homogeneously staining regions. Which of the following genes is most likely to have undergone alterations to produce these findings? □ (A) BCL1 (cyclin gene) □ (B) BCL2 (anti-apoptosis gene) □ (C) IL2 (growth factor gene) □ (D) Lyn (tyrosine kinase gene) □ (E) K-RAS (GTP-binding protein gene) □ (F) N-MYC (transcription factor gene) □ (G) p53 (DNA damage response gene)

5 8 (F) Double minutes and homogeneously staining regions seen on a karyotype represent gene amplifications. Amplification of the N-MYC gene occurs in 30% to 40% of neuroblastomas, and this change is associated with a poor prognosis. The BCL1 and BCL2 genes are mutated in some non-Hodgkin lymphomas. The IL2 mutation may be present in some T cell neoplasms. The Lyn mutation is seen in some immunodeficiency states. K-RAS and p53 mutations are present in many cancers, but not typically childhood neoplasms.

50 A 53-year-old woman sees her physician because she has noticed a change in her bowel habits. On physical examination, there are no abnormal findings, but the test result for stool guaiac is positive. A colonoscopy is performed, and a 3-cm sessile mass is found in the cecum. A biopsy specimen of the mass shows a moderately differentiated adenocarcinoma confined to the mucosa. An abdominal CT scan shows no lymphadenopathy. Given this information, which of the following is the best course of action? □ (A) Perform a limited excision to "shell out" the lesion from its surrounding capsule □ (B) Assume that this represents a metastasis and search for a primary tumor elsewhere □ (C) Resect the tumor and some of the normal surrounding tissue □ (D) Remove the entire colon to prevent a recurrence □ (E) Observe the lesion for further increase in size

50 (C) A malignant neoplasm has a tendency to invade locally. A benign neoplasm is often well circumscribed, although a true capsule is uncommon, and compressed normal surrounding tissue appears to form a discrete border. Such a solitary mucosal lesion is unlikely to represent a metastasis, and the localized lesion can be resected easily. If there is no family history, a familial cancer with high risk of recurrence from multiple polyps is unlikely; local excision is adequate. The biopsy specimen shows a malignant lesion; it must be removed before it increases in size and invades locally or metastasizes.

51 A 50-year-old woman has had easy fatigability and noted a dragging sensation in her abdomen for the past 5 months. Physical examination reveals that she is afebrile. She has marked splenomegaly, but no lymphadenopathy. Laboratory studies show her total WBC count is 250,000/mm3 with WBC differential count showing 68% segmented neutrophils, 11% band neutrophils, 6% metamyelocytes, 5% myelocytes, 5% myeloblasts, 3% lymphocytes, and 2% monocytes. A bone marrow biopsy is performed, and karyotypic analysis of the cells reveals a t(9;22) translocation. Medical treatment with a drug having which of the following modes of action is most likely to produce a complete remission in this patient? □ (A) Antibody binding to ERB-2 receptor □ (B) Inhibiting tyrosine kinase activity □ (C) Selective killing of cells in S phase □ (D) Activating caspases □ (E) Preventing translocation of β-catenin to the nucleus □ (F) Delivering normal p53 into cells with viral vectors

51 (B) This patient has a classic history and t(9;22) translocation with chronic myelogenous leukemia. The translocation causes uncontrolled tyrosine kinase activity of the BCR-ABL fusion gene. These patients undergo remission with drugs that inhibit tyrosine kinases. Antibodies to ERB-2 receptors are beneficial in certain breast tumors with amplification of this gene. Agents that activate caspases theoretically may help in many cases, especially when apoptosis is blocked as in tumors with BCL2 overexpression. Translocation of β-catenin to the nucleus occurs in colon cancers when there is mutational loss of APC genes. Delivery of p53 into cells by viral vectors has not yet been proven to be valuable in cancer treatment, and it is not used in chronic myelogenous leukemia.

54 A 67-year-old man has noted a chronic cough for the past 3 months. On physical examination, there is mild stridor on inspiration over the right lung. A chest radiograph shows a 5-cm right hilar lung mass, and a fine-needle aspiration biopsy specimen of the mass shows cells consistent with squamous cell carcinoma. If staging of this neoplasm is denoted as T2N1M1, which of the following statements is most accurate? □ (A) A CT scan of the head shows a 2-cm right parietal mass □ (B) Serum chemistry shows an elevated corticotropin level □ (C) The mass had infiltrated the chest wall □ (D) The cancer is poorly differentiated □ (E) The tumor is obstructing the left mainstem bronchus

54 (A) The M1 designation indicates that distant metastases are present. Elevated corticotropin levels indicate secretion of an ectopic hormone that may produce a paraneoplastic syndrome. A T2 designation indicates that the overall size of the tumor is not large. The TNM system is used for staging, not grading.

55 A 33-year-old man has experienced occasional headaches for the past 3 months. He suddenly has a generalized seizure. CT scan of the head shows a periventricular 3-cm mass in the region of the right thalamus. A stereotactic biopsy of the mass yields cells diagnostic of a large B cell malignant lymphoma. Which of the following underlying diseases is most likely to be found in this patient? □ (A) Diabetes mellitus □ (B) AIDS □ (C) Hypertension □ (D) Multiple sclerosis □ (E) Tuberculosis

55 (B) Primary or secondary immunodeficiency diseases carry an increased risk of neoplasia, particularly lymphomas. B cell lymphomas of the brain are 1000-fold more common in patients with AIDS than in the general population. Patients with diabetes mellitus can experience various complications, although not neoplasia. Hypertension can lead to central nervous system hemorrhages. Multiple sclerosis is a demyelinating disease and carries no significant risk of neoplasia. Tuberculosis as a chronic infection may lead to amyloidosis, not neoplasia.

56 A 76-year-old man has experienced abdominal pain for the past year. On physical examination, the physician palpates an epigastric mass. An abdominal CT scan shows a 10-cm mass in the body of the pancreas. A needle biopsy specimen of this mass shows a moderately differentiated adenocarcinoma. Mutational analysis of the carcinoma cells shows inactivation of SMAD4 so that transcriptional activation of cyclin-dependent kinases with growth-suppressing effects is diminished. Which of the following regulatory pathways is most likely altered in this man's carcinoma? □ (A) BCL2 □ (B) β-catenin □ (C) MYC □ (D) p53 □ (E) TGF-β

56 (E) TGF-β inhibits cell proliferation by activation of growth-inhibiting genes, such as the CDKIs. All pancreatic cancers and 83% of colon cancers have at least one mutational event in a TGF-β pathway. The BCL2 family of genes acts as regulators of apoptosis. The β-catenin pathway seen with the APC gene is involved with growth regulation; loss of the APC gene loci leads to failure in destruction of β-catenin, which translocates to the cell nucleus, where it functions as a transcription factor promoting growth. The MYC gene is a target of the activated RAS pathway. The p53 protein is involved in tumor suppression.

59 A 42-year-old man is concerned about a darkly pigmented "mole" on the back of his hand. The lesion has enlarged and bled during the past month. On physical examination, there is a slightly raised, darkly pigmented, 1.2-cm lesion on the dorsum of the right hand. The lesion is completely excised. Microscopically, a malignant melanoma is present. Which of the following factors presents the greatest risk for the development of this neoplasm? □ (A) Smoking tobacco □ (B) Ultraviolet radiation □ (C) Chemotherapy □ (D) Asbestos exposure □ (E) Allergy to latex

59 (B) Worldwide, increasing numbers of skin cancers occur because of sun exposure. The ultraviolet light damages the skin and damages cellular DNA, leading to mutations that escape cellular repair mechanisms. Smoking tobacco is related to many cancers, but skin cancers are not typically associated with this risk factor. Chemotherapeutic agents have carcinogenic potential, particularly alkylating agents such as cyclophosphamide, but leukemias and lymphomas are the usual result. Asbestos exposure increases lung carcinoma risk in smokers and can lead to rare mesotheliomas of pleura. Allergic reactions do not promote cancer.

6 A 19-year-old woman goes on a camping trip to a wooded area with lots of insects, but has forgotten to bring insect repellant. A month later, she has increasing malaise, low-grade fever, headaches, and myalgias. On physical examination, she has hepatosplenomegaly. Laboratory studies show hemoglobin, 10.4 g/dL; WBC count, 5820/mm3; and platelet count, 205,000/mm3. Her peripheral blood smear is shown in the figure. Which of the following infectious agents is most likely to produce these findings? □ (A) Babesia microti □ (B) Borrelia burgdorferi □ (D) Entamoeba histolytica □ (E) Giardia lamblia □ (F) Rickettsia rickettsii □ (G) Wuchereria bancrofti

6 (A) This patient's travel history suggests an insect-borne disease. The figure shows the characteristic tetrad and ring forms of babesiosis, which is an uncommon malaria-like protozoan disease. The northeastern United States is an endemic area. The vector is the deer tick, just as with Lyme disease from Borrelia burgdorferi, which is a spirochete. Amebiasis produces colitis with diarrhea. Giardiasis typically produces self-limited, watery diarrhea. Rickettsia rickettsii causes Rocky Mountain spotted fever, which occurs sporadically in the United States in areas other than the Rocky Mountains and produces signs and symptoms from damage to vascular endothelium and smooth muscle similar to a vasculitis. Wuchereria bancrofti is a form of filariasis that can cause elephantiasis, owing to lymphatic obstruction in the presence of an inflammatory reaction to the adult filarial worms.

6 A 20-year-old man has a raised, irregular, pigmented lesion on his forearm that has increased in size and become more irregular in color over the past 4 months. Physical examination shows a 0.5 × 1.2 cm black-to-brown lesion with irregular borders. An excisional biopsy specimen shows a malignant melanoma that extends into the reticular dermis. Family history indicates that the patient's paternal uncle died of metastatic melanoma that spread to the liver after excision of a primary lesion on the foot. His grandfather required enucleation of the left eye because of a "dark brown" mass in the eyeball. Which of the following genes is most likely to have undergone mutation to produce these findings? □ (A) BCL2 (anti-apoptosis gene) □ (B) C-MYC (transcription factor gene) □ (C) IL2 (growth factor gene) □ (D) Lyn (tyrosine kinase gene) □ (E) p16 (cell cycle inhibition) □ (F) p53 (DNA damage response gene) □ (G) PDGF (growth factor overexpression)

6 (E) This patient has a family history of malignant melanoma. Familial tumors often are associated with inheritance of a defective copy of one of several tumor suppressor genes. In the case of melanomas, the implicated gene is called p16, or INK4a. The product of the p16 gene is an inhibitor of cyclin-dependent kinases. With loss of control over cyclin-dependent kinases, the cell cycle cannot be regulated, favoring neoplastic transformation. BCL2 is present in some lymphoid neoplasms. The C-MYC gene is mutated in various carcinomas, but is not known to be specifically associated with melanomas. The IL2 mutation is associated with some T cell neoplasms. The Lyn mutation is seen in some immunodeficiency states. p53 mutations occur in many cancers, but not specifically in familial melanomas. PDGF can be overexpressed in some central nervous system gliomas and some osteosarcomas.

60 A clinical study involves patients diagnosed with carcinoma whose tumor stage is T4N1M1. The patients' survival rate 5 years from the time of diagnosis is less than 50%, regardless of therapy. Which of the following clinical findings is most likely to be characteristic of this group of patients? □ (A) Cachexia □ (B) Cardiac murmur □ (C) Icterus □ (D) Loss of sensation □ (E) Splenomegaly □ (F) Tympany

60 (A) Cachexia is a common finding in advanced cancers, and weight loss without dieting in an adult is a "red flag" for malignancy. The exact cause for this is unknown, but increases in circulating factors such as tumor necrosis factor may play a role. Cardiac murmurs may occur in the development of nonbacterial thrombotic endocarditis, a feature of a hypercoagulable state that may occur with advanced malignancies. Icterus is most likely to occur when there is obstruction of the biliary tract by a mass (e.g., as in pancreatic cancer), but metastases are unlikely to cause such an obstruction. Neurologic abnormalities may occur in local tumor growth impinging on nerves, but dull constant pain is the most likely abnormality in malignant neoplasms that invade nerves. Metastases to the spleen are uncommon. Tympany is uncommon in cancer because obstruction by a mass tends to be incomplete and develop over a long time. (Hint: an empty beer keg is tympanitic when percussed.)

61 A 53-year-old man diagnosed with oral cancer and treated with radiation and chemotherapy 1 year ago now has a positron emission tomography (PET) scan of his neck that shows a single focus of increased uptake. This focus is resected and microscopic examination shows that it is a metastasis. Molecular analysis of this cancer shows p53, PTEN, and c-MYC gene mutations. Which of the following metabolic pathways is most likely upregulated to promote his cancer cell survival and proliferation? □ (A) Aerobic glycolysis □ (B) Gluconeogenesis □ (C) Hexose monophosphate shunt □ (D) Oxidative phosphorylation □ (E) Purine degradation

61 (A) The PET scan is based upon selective uptake of a glucose derivative into tumor cells. The Warburg effect occurs when cancer cells shift their metabolism to aerobic glycolysis for selective growth advantage under harsh circumstances. Glycolysis also yields pyruvate for anabolic demands of increased tumor doublings. The p53 and c-MYC genes favor this metabolic change, while PTEN inhibits tumor cell autophagy, giving cancer cells an edge in growth. Cancer cells are less differentiated than normal cells and thus have decreased ability to do many complex biochemical processes, so they favor a simple one—glycolysis. Gluconeogenesis is a function of hepatocytes in response to decreased caloric intake. The HMP shunt and Krebs cycle are more useful to normal cells maintaining themselves at the status quo. Neoplasms generate large amounts of purines from cell divisions and cell turnover that must be eliminated as uric acid, but neoplastic cells do not perform this task.

7 A 25-year-old soldier stationed in the Middle East has noted the appearance of a 0.5-cm papule on his left forearm. It becomes a 1-cm nodule with a central depression, then ulcerates over the next month. On physical examination, the 2-cm Robbins & Cotran Review of Pathology Pg. 126 ulcerated lesion has an indurated border, and there are three smaller satellite lesions. There is no hepatosplenomegaly, but he has left axillary lymphadenopathy. Laboratory studies show hemoglobin, 14.1 g/dL; hematocrit, 42.5%; platelet count, 233,200/mm3; and WBC count, 6270/mm3. Which of the following infectious organisms is most likely to produce these findings? □ (A) Borrelia recurrentis □ (B) Brugia malayi □ (C) Leishmania major □ (D) Listeria monocytogenes □ (E) Mycobacterium leprae □ (F) Plasmodium falciparum □ (G) Trypanosoma gambiense

7 (C) This patient has cutaneous leishmaniasis, and the original papule was at the site of the sand fly vector bite. Leishmaniasis is endemic in the Middle East, South Asia, Africa, and Latin America. The organisms proliferate within macrophages in the mononuclear phagocyte system and can cause regional lymphadenopathy. The cutaneous form does not have bone marrow involvement and splenic enlargement, so pancytopenia is not present. Borreliosis causes relapsing fever and is transmitted via body lice. Brugia malayi is a nematode transmitted by mosquitoes that leads to filariasis involving lymphatics to produce elephantiasis. Leishmania donovani is transmitted by sand flies and leads to infection of macrophages, which produces hepatosplenomegaly, lymphadenopathy, and bone marrow involvement with pancytopenia. Listeriosis is most often acquired via contaminated food or water. In most adults, it produces mild diarrheal illness, but in some adults and children, and in fetuses, it may produce meningitis or dissemination with microabscess (microgranuloma) formation. Mycobacterium leprae causes Hansen's disease, with infection of peripheral nerves and skin. In individuals with a strong immune response, the tuberculoid form of this disease results in granuloma formation; in individuals with a weak immune response, the lepromatous form occurs, characterized by large numbers of macrophages filled with short, thin, acid-fast bacilli. Malaria, caused by Plasmodium falciparum, produces hemolytic anemia, splenomegaly, and cerebral thrombosis. African trypanosomiasis produces sleeping sickness.

7 A 32-year-old woman has experienced dull pelvic pain for the past 2 months. Physical examination shows a right adnexal mass. An abdominal ultrasound scan shows a 7.5-cm cystic ovarian mass. The mass is surgically excised. The surface of the mass is smooth, and it is nonadherent to surrounding pelvic structures. On gross examination, the mass is cystic and filled with hair. Microscopically, squamous epithelium, tall columnar glandular epithelium, cartilage, and fibrous connective tissue are present. Which of the following is the most likely diagnosis? □ (A) Adenocarcinoma □ (B) Fibroadenoma □ (C) Glioma □ (D) Hamartoma □ (E) Mesothelioma □ (F) Rhabdomyosarcoma □ (G) Teratoma

7 (G) A teratoma is a neoplasm derived from totipotential germ cells that differentiate into tissues that represent all three germ layers: ectoderm, endoderm, and mesoderm. When the elements all are well differentiated, the neoplasm is "mature" (benign). Adenocarcinomas have malignant-appearing glandular elements. Fibroadenomas have a benign glandular and stromal component; they are common in the breast. Gliomas are found in the central nervous system. Hamartomas contain a mixture of cell types common to a tissue site; the lung is one site for this uncommon lesion. A mesothelioma arises from the lining of thoracic and abdominal body cavities. A rhabdomyosarcoma comprises cells that poorly resemble striated muscle; most arise in soft tissues.

8 A 50-year-old woman comes to the health center because she has observed a small vesicle on her right labium majus. She is sexually active. On physical examination, tender inguinal lymph nodes are palpable. She was diagnosed with lymphoma 10 years ago. A biopsy of one of the lymph nodes is performed to exclude malignancy. Histologically, the biopsy specimen shows multiple abscesses in which central necrosis is surrounded by palisading histiocytes. This morphology, combined with the clinical picture, is most likely to be a complication of which of the following conditions? □ (A) Chlamydia trachomatis cervicitis □ (B) Herpes simplex virus infection of the perineum □ (C) Recurrent non-Hodgkin lymphoma □ (D) Candida albicans vaginitis □ (E) Gardnerella vaginalis vaginosis

8 (A) Infection with Chlamydia trachomatis is a common sexually transmitted disease. Most cases produce only urethritis and cervicitis; however, some strains of C. trachomatis can produce lymphogranuloma venereum, a chronic ulcerative disease that is more endemic in Asia, Africa, and the Caribbean. In this disease, there is a mixed granulomatous and neutrophilic inflammatory reaction, as seen in this patient. In contrast, herpes simplex virus produces clear mucocutaneous vesicles with no exudates and is unlikely to involve lymph nodes. Recurrent lymphoma is characterized by sheets or nodules of pleomorphic lymphocytes without significant inflammation. Candidiasis can produce superficial inflammation with an exudate, but it is rarely invasive or disseminated in non-immunosuppressed individuals. Bacterial vaginosis due to Gardnerella produces a whitish discharge that has a "fishy" odor.

8 A 30-year-old man has a 15-year history of increasing numbers of multiple benign skin nodules. On physical examination, the firm nodules average 0.5 to 1 cm and appear to be subcutaneous. Further examination shows numerous oval 1- to 5-cm pigmented skin lesions. Ophthalmoscopic examination shows hamartomatous nodules on the iris. A biopsy specimen of one skin nodule shows a neoplasm that is attached to a peripheral nerve. Which of the following mechanisms of transformation is most likely related to the mutation that this patient has inherited? □ (A) Persistent activation of the RAS gene □ (B) Increased production of epidermal growth factor □ (C) Decreased susceptibility to apoptosis □ (D) Impaired functioning of mismatch repair genes □ (E) Inactivation of the RB gene

8 (A) This patient has clinical features of neurofibromatosis type 1. The NF1 gene encodes a GTPase-activating protein that facilitates the conversion of active (GTP-bound) RAS to inactive (GDP-bound) RAS. Loss of NF1 prevents such conversion and traps RAS in the active signal-transmitting stage. All other listed mechanisms also are involved in carcinogenesis, but in different tumors.

9 A 31-year-old man has had increasing respiratory difficulty for the past 2 days. On physical examination, crackles are auscultated over all lung fields. A chest radiograph shows bilateral interstitial infiltrates. Laboratory studies show that the patient is HIV-positive and has a plasma titer of 26,800 copies of HIV-1 RNA/mL. A transbronchial biopsy is performed; the microscopic appearance of the specimen is shown in the figure. On the basis of the clinical and histologic findings, which of the following is the most likely causative organism of this acute illness? □ (A) Epstein-Barr virus □ (B) Cytomegalovirus □ (C) Respiratory syncytial virus □ (D) Herpes zoster virus □ (E) Adenovirus

9 (B) This patient has high HIV-1 RNA levels that are consistent with the diagnosis of AIDS. Although patients with AIDS are susceptible to many microbes, infections with cytomegalovirus are particularly common. The biopsy specimen shows an enlarged cell containing a distinct intranuclear inclusion and ill-defined cytoplasmic inclusions, which are typical of cytomegalovirus infection. Epstein-Barr virus infection is seen frequently in patients with HIV infection, but there are no distinct pulmonary lesions associated with it. Respiratory syncytial virus infections are seen in children, but rarely in adults. Herpes zoster infections are most likely to affect the peripheral nervous system, rarely can become disseminated to affect the lungs in immunosuppressed patients, and produce a different appearance than that shown. Adenovirus is a common viral pathogen in adults that may produce a clinically significant pneumonia, and intranuclear inclusions may be present, but the cells are not large, and cytoplasmic inclusions are absent.

9 A study of patients treated with chemotherapy protocols for cancer shows that 10% of them subsequently develop a second cancer, a much higher incidence compared with a control group not receiving chemotherapy. These chemotherapy protocols included the alkylating agent cyclophosphamide. What is the most likely mechanism of action by this alkylating agent for development of the subsequent carcinomas in these treated cancer patients? □ (A) Activation of protein kinase C □ (B) Activation of endogenous viruses □ (C) Blockage of TGF-β pathways □ (D) Direct DNA damage □ (E) Inhibition of DNA repair □ (F) Inhibition of telomerase

9 (D) Chemical carcinogens have highly reactive electrophile groups that can directly damage DNA, leading to mutations. Direct-acting agents, such as alkylating chemotherapy drugs, do not require conversion to a carcinogen. Some agents, such as polycyclic hydrocarbons, require metabolic conversion to a carcinogen and are called indirect-acting agents. Phorbol esters are examples of promoters of chemical carcinogenesis that cause tumor promotion by activating protein kinase C. This enzyme phosphorylates several substrates in signal transduction pathways, including those activated by growth factors, and the cells divide. Forced cell division predisposes the accumulation of mutations in cells previously damaged by exposure to a mutagenic agent (initiator). The TGF-β pathways work via growth inhibition. Proteins such as p53 that function in DNA repair pathways can become nonfunctional through mutation. Viral infections such as hepatitis B and C tend to promote growth by binding to p53 and inactivating its protective function. Telomerase activity is not affected by carcinogens.


Related study sets

CH 12: The Correlational Research Strategy

View Set

Chapter 08 Capitalizing on Global and Regional Integration

View Set

Chapter 6- Types of Business Ownership

View Set